You are on page 1of 69

2002 Self-Assessment Exercise XXIV. Adolescent medicine and gynecology [Return to Category List] Questions Question 51. Answer.

A 16-year-old girl comes to your school-based clinic complaining of lower abdominal pain, nausea, and vomiting. She denies vaginal bleeding or discharge. Her last episode of unprotected sexual intercourse was 1 month ago. Physical examination reveals a temperature of 101F (38.5C), moderate lower abdominal pain, mild cervical motion tenderness, and unilateral adnexal tenderness. A urine pregnancy test is negative. Of the following, the MOST appropriate management for this adolescent is: A. B. C. D. E. hospitalization for further evaluation and treatment outpatient treatment with oral antibiotic therapy reassurance and re-evaluation in 48 hours referral to a gynecologist for laparoscopy referral to a radiologist for pelvic ultrasonography

Question 183. Answer. You are the guest speaker at a local high school health class. The subject is reproductive health, and a student asks you a question about oral contraceptives. Her grandmother told her that they are especially harmful to a young woman's health. A TRUE statement regarding adolescents and currently available contraceptive methods is that: A. all methods of contraception are associated with fewer health risks than pregnancy and delivery B. compliance with most methods is equal between adolescents and adults in the United States C. condoms and spermicides provide protection against all sexually transmitted diseases D. E. oral contraceptives significantly reduce the risk of breast cancer the most common side effect of an injectable progestin is alopecia

Answers

Critique 51.

Preferred Response: A

[View Question] The adolescent girl described in the vignette displays classic signs and symptoms of pelvic inflammatory disease (PID), an inflammatory process of the upper female genital tract that may include any combination of the following: endometritis, salpingitis, tubo-ovarian abscess, and pelvic peritonitis. Other symptoms of PID are vaginal discharge, irregular vaginal bleeding, dysuria, dyspareunia, fever, and diarrhea. Patients also may present with right upper quadrant pain secondary to perihepatitis (Fitz-HughCurtis syndrome). Physical examination may reveal vaginal or cervical discharge or mucopurulent cervicitis in addition to cervical motion and adnexal tenderness. Rebound tenderness and guarding of the abdomen also may be present. Current recommended treatment regimens for PID are aimed at the early detection and aggressive treatment of presumptive infections, especially in high-risk individuals such as adolescents. Several studies have confirmed that untreated and indolent cases of PID are significant causes of future infertility and ectopic pregnancy in women. Therefore, minimum criteria have been established for the presumptive diagnosis of PID to justify initiating early therapy. Because PID is a polymicrobial infection, both inpatient and ambulatory treatment regimens include antimicrobial coverage for anaerobic and aerobic bacteria. Associated endocervical infections with Neisseria gonorrhoeae occur in up to 75% of cases of PID; Chlamydia trachomatis cervicitis may occur in up to 45% of cases. Although indications for the hospitalization of an adolescent who has PID are controversial, it is the most appropriate management for the girl described in the vignette because she is febrile and experiencing nausea and vomiting. In addition, current treatment strategies (Figure 51A) advocate early presumptive therapy. Other indications for the inpatient treatment of PID include medical instability, peritoneal signs, and intolerance of oral medications. Most teens are not treated as outpatients simply because of their questionable compliance with outpatient medications and follow-up. Referral to a gynecologist or radiologist is not indicated because the patient is not manifesting any of the complications associated with PID, such as a tubo-ovarian abscess. Reassurance and reevaluation is inappropriate; treatment for PID is immediate antibiotic therapy. References: Centers for Disease Control and Prevention. 1998 guidelines for treatment of sexually transmitted diseases. MMWR Morb Mortal Wkly Rep. 1998;47(RR-1):1-111 Lappa S, Moscicki AB. The pediatrician and the sexually active adolescent. A primer for sexually transmitted diseases. Pediatr Clin North Am. 1997;44:1405-1445 Pletcher JR, Slap GB. Pelvic inflammatory disease. Pediatr Rev. 1998;19:363-367 Critique 183. Preferred Response: A

[View Question] All methods of contraception are associated with fewer health risks than pregnancy and delivery, including overall mortality. Especially among adolescents, who generally are healthy, the occurrence of adverse side effects with most forms of contraception is minimal. For example, the risk of a thromboembolic event in an otherwise healthy teenager taking oral contraceptives is negligible compared with her risk of developing a complication associated with an unplanned pregnancy, such as pregnancy-induced hypertension. Unfortunately, the public misconception is that contraception is dangerous, ineffective, and too costly for most young women to obtain. Current educational programs for young adolescents, therefore, are aimed at disproving these beliefs. Compliance with most methods of contraception is worse in adolescents compared to adults in the United States. Misinformation and a sense of invulnerability (the belief that I am sure I wont get pregnant) probably contribute to noncompliance. Condoms and spermicides provide protection against most but not all sexually transmitted diseases. Human papillomavirus and herpes simplex virus still can be acquired if the partners lesions are located at the base of the penis or on the perineum. In addition, condoms may break, and non-latex condoms may allow passage of viral particles (eg, human immunodeficiency virus). Oral contraceptives do not affect a womans risk for breast cancer, an issue that was more controversial in the past. Although hair loss occasionally is a side effect of injectable progestin therapy, the most common side effect is a change in menstrual patterns, namely, prolonged and irregular bleeding. References: Brown RT, Cromer BA. The pediatrician and the sexually active adolescent. Sexual activity and contraception. Pediatr Clin North Am. 1997;44:1379-1390 Stevens-Simon C. Providing effective reproductive health care and prescribing contraceptives for adolescents. Pediatr Rev. 1998;19:409-417 2001 Self-Assessment Exercise XXIV. Adolescent medicine and gynecology [Return to Category List] Questions [Print Directions] Question 3. Answer.

At a 10-year-old girl's health supervision visit, her mother asks at what age her daughter will reach full physical maturity. Of the following, your MOST appropriate response is that

A.

peak height velocity among females occurs during the last year of puberty

B. pubertal progression from Sexual Maturity Rating (Tanner) stage 2 to 5 can take up to 5 years C. D. E. puberty usually is complete with the onset of menarche puberty usually is complete within 2 years of thelarche the duration of puberty cannot be predicted Answer.

Question 18.

A 10-year-old girl is brought to your office for evaluation of bloody discharge that her mother noted on the girl's underpants for 2 days. She denies any history of trauma or sexual molestation. She takes no medications. Findings on physical examination include bilateral symmetric breast buds, sparse downy pubic hair, and a small amount of a brown vaginal discharge. Of the following, the MOST likely explanation for these findings is A. B. C. D. E. a vaginal foreign body menarche nonspecific vaginitis thrombocytopenia urethral prolapse Answer.

Question 33.

You are seeing a 10-year-old girl for a health supervision visit. Her mother asks how much of an increase in her daughter's height to expect annually and whether this amount will change during puberty. Of the following, the height velocities that are MOST likely to occur are

Average Prepubertal A. B. C. D. E. 1 to 2 cm/y 3 to 4 cm/y 5 to 6 cm/y 7 to 8 cm/y 9 to 10 cm/y

Adolescent Growth Spurt

5 to 6 cm/y 7 to 8 cm/y 9 to 10 cm/y 11 to 12 cm/y 13 to 14 cm/y

Question 50.

Answer.

A 15-year-old girl is worried because she has not begun menstruating. Breast growth began at age 14. Her mother and sister began menstruating at age 12. The girl's weight is at the 25th percentile for age, and her height is below the 5th percentile. Small breast buds are noted on physical examination, but there is no pubic hair. Of the following, diagnostic studies are MOST likely to show A. B. C. D. E. abnormal karyotyping decreased serum testosterone decreased thyroid stimulating hormone elevated erythrocyte sedimentation rate elevated serum prolactin Answer.

Question 65.

The MOST prevalent sexually transmitted infection among adolescents and young adults in the United States is A. B. C. D. E. chlamydia genital herpes gonorrhea human papillomavirus syphilis Answer.

Question 81.

An otherwise healthy 10-year-old girl is brought to your office for evaluation of unilateral breast tenderness without galactorrhea. Physical examination reveals bilateral breast buds that measure approximately 2 cm in diameter. She has no axillary or pubic hair. The Sexual Maturity Rating (Tanner) stage of this girl's tissue is stage A. B. C. D. E. 1 2 3 4 5

Question 96.

Answer.

A 12-year-old premenarcheal girl has had a persistent vaginal discharge for 3 months. Physical examination findings include Sexual Maturity Rating (Tanner) stage 3 breasts and genitalia. The hymen is annular and smooth. A thin, white, odorless discharge is noted at the introitus. Microscopic analysis of the fluid reveals vaginal epithelial cells, rod-shaped organisms, and few white blood cells. Of the following, the MOST appropriate treatment for this girl is A. B. C. D. E. azithromycin orally cefixime orally clotrimazole cream vaginally metronidazole orally reassessment in 2 to 3 months

Question 113. Answer. An 8-year-old girl is diagnosed as having precocious puberty. Her mother asks you what, if any, effect this will have on her growth and development. Of the following, your MOST likely response is that A. administration of growth hormone will be necessary when the girl completes puberty B. C. D. E. early puberty will have no effect on immediate psychological development final adult height primarily is determined genetically radiographs of the hand and wrist can help predict final adult height relatively early puberty often results in short adult height

Question 129. Answer. You are evaluating a 15-year-old boy for short stature. As part of your evaluation, you order a radiograph of the hand and wrist. Of the following, the MOST appropriate use of this diagnostic study is to A. B. C. D. determine pubertal stage evaluate skeletal maturity predict adult height screen for chronic illness

E.

verify that puberty has begun

Question 144. Answer. You are evaluating a healthy 16-year-old boy for short stature. His growth chart reveals height measurements following the 5th percentile and weight measurements following the 25th percentile. Findings on physical examination include Sexual Maturity Rating (Tanner) stage 2 pubic hair and stage 3 genitalia. You diagnose constitutional delay of growth and adolescence. Radiographic studies MOST likely would show a bone age A. B. C. D. E. equal to the chronologic age greater than the chronologic age greater than the height age less than the chronologic age less than the height age

Question 161. Answer. A 13-year-old girl complains that her menses are irregular and can last for as long as 7 days. Menarche occurred at 12 years of age. She takes no medications, and her medical history is unremarkable. Her breasts and pubic hair are at Sexual Maturity Rating (Tanner) stage 4. Laboratory evaluation reveals a negative urine pregnancy test, a normal thyroid-stimulating hormone level, and a hemoglobin of 110 g/L (11 g/dL). Of the following, the MOST appropriate next step in management is to A. B. C. D. E. administer oral estrogen supplementation administer oral ferrous sulfate obtain a serum von Willebrand panel obtain pelvic ultrasonography refer to a gynecologist for further evaluation

Question 177. Answer. You are evaluating an otherwise healthy 14-year-old boy for short stature. His weight is at the 50th percentile for age, and his height is at the 5th percentile. Physical examination of his genitalia reveals a slightly enlarged penis and testes, with light pubic hair laterally. Of the following, the MOST appropriate next step is A. intramuscular androgens

B. C. D. E.

intramuscular growth hormone oral thyroxine radiography of the hand and wrist reassurance and re-evaluation in 6 months

Question 193. Answer. A mother is concerned that her 11-year-old daughter has had a persistent vaginal discharge for the past 4 months. Wet mount shows multiple vaginal epithelial cells and a few white blood cells. Breast development began at age 9. Her breast and pubic hair are at Sexual Maturity Rating (Tanner) stage 3. Of the following, this vaginal discharge is MOST likely to A. B. C. D. E. be accompanied by dysuria cause significant vulvar pruritus have a "cottage cheese" appearance have a "fishy", amine odor precede menses by 3 to 6 months

Question 208. Answer. The normal range of hemoglobin concentration in boys begins to increase substantially compared with girls at A. B. C. D. E. birth beginning of puberty early prenatal development end of puberty toddler years

Question 225. Answer. A 12-year-old girl presents with a nonspecific vaginal discharge of 3 months' duration. She denies a history of sexual activity or molestation. Findings on physical examination include breasts that are at Sexual Maturity Rating (SMR) (Tanner) stage 4; pubic hair at SMR 3; and a thin, white, opaque discharge at the introitus. The pH of the vaginal fluid is 4.0. Of the following, the MOST likely cause of this girl's vaginal discharge is A. bacterial vaginosis

B. C. D. E.

Candida albicans Neisseria gonorrhoeae physiologic leukorrhea trichomoniasis

Question 240. Answer. A 14-year-old girl complains of heavy, irregular menstrual periods that last 8 to 10 days. Menarche occurred 1 year ago. She takes no medications, and findings on physical examination are unremarkable. Laboratory evaluation reveals a hemoglobin of 100 g/L (10.0 g/dL). Of the following, the MOST appropriate treatment for this patient is A. B. C. D. E. dilatation and curettage intravenous conjugated estrogen oral contraceptive pills reassurance that her menses are normal whole blood transfusion

Question 255. Answer. During a health supervision visit, a 14-year-old boy expresses concern that he is the shortest person in his class. He has no chronic illnesses, and his height has been following the 10th percentile. Physical examination reveals Sexual Maturity Rating (Tanner) stage 2 pubic hair and stage 3 genitalia. Of the following, the MOST appropriate management of this adolescent is to A. B. C. D. E. administer growth hormone administer testosterone obtain a magnetic resonance imaging scan of the head obtain a serum thyroid hormone level schedule a re-evaluation in 4 to 6 months

Answers Critique 3. Preferred Response: B

[View Question]

Pubertal progression from Sexual Maturity Rating (SMR) (Tanner) stage 2 to stage 5 can require 2.5 to 5 years to complete, although this varies among individuals. This range can be even longer in some females, in whom up to 8 years has been reported. In both males and females, puberty begins with the attainment of SMR 2, usually at 11 to 11.5 years of age. The remainder of puberty involves growth of the testes, phallus, epididymis, prostate, and pubic hair in males and development of breasts and pubic hair in females. The typical sequence of pubertal events differs significantly between males and females. The most notable difference is the later occurrence of peak height velocity (PHV) in males in relation to sexual development than in females. In the average adolescent female, the growth spurt begins approximately 1 year before breast development, but PHV does not occur until 1 year later, which corresponds to SMR stage 2. In contrast, PHV occurs in males after SMR stage 4, which often is during the last 1 to 2 years of puberty. In girls, puberty is complete after SMR 5 is attained, not with the onset of menarche or within 2 years of thelarche. The duration of puberty often can be predicted based on data for an average adolescent coupled with an individuals family history. References: Kreipe RE. Normal somatic adolescent growth and development. In: McAnarney ER, Kreipe RE, Orr DP, Comerci GD, eds. Textbook of Adolescent Medicine. Philadelphia, Pa: WB Saunders Co; 1992:44-67 Kulin HE, Muller J. The biological aspects of puberty. Pediatr Rev. 1996;17:75-86 Neinstein LS, Kaufman FR. Normal physical growth and development. In: Neinstein LS, ed. Adolescent Health Care: A Practical Guide. 3rd ed. Baltimore, Md: Williams & Wilkins; 1996:3-39 Critique 18. Preferred Response: A

[View Question] The development of secondary sexual characteristics progresses in a sequential and orderly fashion in most girls. Beginning with the onset of the growth spurt and the development of breast buds, most females experience the onset of puberty at age 11. This is followed by the appearance of fine, downy pubic hair. The peak height velocity occurs an average of 1 year later, followed by further development of breast tissue and coarser pubic hair over the next few years. The timing of menarche also is fairly consistent. It usually occurs after the adolescent attains Sexual Maturity Rating (SMR) (Tanner) stage 3 or 4. Approximately 25% of girls experience menarche in SMR stage 3 and 60% in SMR stage 4. Within 1 year of attaining SMR stage 5, 95% of girls have begun menstruating. In contrast, only 5% of girls in SMR stage 2 have menarche.

Accordingly, it is unlikely that the girl described in the vignette is experiencing menarche because findings on physical examination are consistent with SMR stage 2. Although a vaginal discharge often accompanies a nonspecific vaginitis, it is usually yellowish. A brown, bloody discharge frequently is found with the presence of a foreign body (Figure 18A) in the vagina. In most young girls, the foreign body is comprised of small pieces of toilet paper; in an older adolescent, it could be a retained tampon. None of the findings on history or physical examination of the girl described in the vignette is suggestive of thrombocytopenia. Urethral prolapse (Figure 18B) appears as an annular, donut-shaped, erythematous swelling or protrusion from the urethral meatus. Eversion of the urethral mucosa leads to a bloody discharge. References: Emans SJ. Vaginal bleeding. In: Emans SJH, Laufer MR, Goldstein DP, eds. Pediatric and Adolescent Gynecology. 4th ed. Philadelphia, Pa: Lippincott-Raven; 1998:93-107 Emans SJ. Vulvovaginal problems in the prepubertal child. In: Emans SJH, Laufer MR, Goldstein DP, eds. Pediatric and Adolescent Gynecology. 4th ed. Philadelphia, Pa: LippincottRaven; 1998:75-107 Critique 33. Preferred Response: C

[View Question] Prior to the onset of puberty, the average height velocity in both males and females is 5 to 6 cm/y. During puberty, the height velocity accelerates and peaks during the adolescent growth spurt, which lasts 24 to 36 months. Peak height velocity (PHV) occurs when linear growth is at maximal speed. PHV in the normal female occurs about 2 years after the initiation of the growth spurt and averages approximately 9 cm/y (range, 5.5 to 11 cm/y). This maximal growth rate occurs 6 to 12 months prior to the onset of menses and lasts for only a few months. There is a deceleration in linear growth over the next 2 years, and epiphyseal closure occurs approximately 2 years after menarche. In normal males, the PHV averages approximately 10 cm/y (range, 6 to 13 cm/y). The beginning of their growth spurt and their PHV lag behind that of girls. However, as linear growth rate begins to decelerate in girls, the growth rate in boys begins to accelerate. Although PHV only occurs for a few months, males are ultimately taller than females because they have a greater PHV during their growth spurt and experience approximately 2 more years of prepubertal growth at an average height velocity of 5 to 6 cm/y. References: Kreipe RE. Normal somatic adolescent growth and development. In: McAnarney ER, Kreipe RE, Orr DP, Comerci GD, eds. Textbook of Adolescent Medicine. Philadelphia, Pa: WB Saunders Co; 1992:44-67

Neinstein LS, Kaufman FR. Normal physical growth and development. In: Neinstein LS, ed. Adolescent Health Care: A Practical Guide. 3rd ed. Baltimore, Md: Williams & Wilkins; 1996:3-39 Critique 50. Preferred Response: A

[View Question] Primary amenorrhea is defined as the absence of menstruation by age 16 in the presence of normal breast and pubic hair development. However, lack of menses by age 14 in the absence of normal development of breasts and pubic hair also warrants evaluation. Additional guidelines that are less dependent on the chronologic age of the patient also can be used to evaluate pubertal delay and amenorrhea. They include the absence of breast budding by age 13 and the failure to achieve menarche within 2 years of completing Sexual Maturity Rating (SMR) (Tanner) stage 4 breast and pubic hair development. The adolescent described in the vignette clearly fits the criteria for pubertal delay and, therefore, warrants an evaluation. Although breast growth began 1 year ago, she only has reached SMR stage 2 breast development and still has no pubic hair (SMR stage 1). Also of note is her short stature, with the height below the 5th percentile. Among other etiologies, gonadal dysgenesis or Turner syndrome must be considered in girls who have pubertal delay and amenorrhea, especially if it is accompanied by short stature, as seen for the girl described in the vignette. Approximately 50% of individuals who have Turner syndrome exhibit mosaicism or a structural abnormality of the X chromosome. Because of the wide genotypic and phenotypic variation, some patients may present with secondary amenorrhea due to ovarian failure or may have normal menses and be fertile. Other clinical manifestations of Turner syndrome include short stature, broad chest, webbed neck, remnant gonadal streaks, hypoestrogenism, sexual immaturity, and amenorrhea. Pubic hair progression to SMR stage 2 or 3 often is in response to adrenal androgens rather than ovarian hormone. The laboratory evaluation of an adolescent who has primary amenorrhea and pubertal delay should include a karyotype, complete blood count, erythrocyte sedimentation rate, and thyroid studies. Serum levels of prolactin, follicle-stimulating hormone, and luteinizing hormone also should be obtained. Measurement of serum testosterone is not indicated in the evaluation of a patient who has pubertal delay and amenorrhea. Diagnostic studies in Turner syndrome most likely will show an abnormal karyotype. Results of thyroid studies, serum prolactin concentrations, and the erythrocyte sedimentation rate will be normal. References: Braverman PK, Soundheimer SJ. Menstrual disorders. Pediatr Rev. 1997;18:17-26

Iglesias EA, Coupey SM. Menstrual cycle abnormalities: diagnosis and management. Adolesc Med. 1999;10:255-273 Pletcher JR, Slap GB. Menstrual disorders. Amenorrhea. Pediatr Clin North Am. 1999;46:505518 Critique 65. Preferred Response: D

[View Question] Infection with human papillomavirus (HPV) is the most prevalent sexually transmitted disease (STD) among adolescents and young adults. Although only about 1% of sexually active individuals in this age group have genital warts (Figure 65A), as many as 46% of asymptomatic young women have evidence of HPV infection when sensitive diagnostic techniques such as polymerase chain reaction are employed. These high rates of infection are concerning, because HPV is associated with cervical intraepithelial neoplasia. There is evidence, however, that asymptomatic HPV infection is not persistent in most young women. In a study of 608 college students, for example, the cumulative incidence of infection over 3 years was 43%, but the median duration of infection was 8 months, and only 9% were infected after 24 months. Chlamydia trachomatis is the most common bacterial STD, responsible for more than 4 million infections annually in the United States. Rates of infection are highest in adolescents and are two to three times greater than for gonorrhea. Among adolescent females attending urban clinics for gynecologic care, the prevalence of infection typically has ranged from 13% to 26%. Studies of asymptomatic sexually active males indicate that 5% to 10% are infected. Comprehensive clinical surveillance data for genital herpes simplex virus (HSV) infections do not exist and, therefore, the prevalence of HSV infection often is derived from serologic studies in which antibodies to HSV-2 are measured. A comparison of data from the National Health and Nutrition Examination Surveys II (1972 to 1980) and III (1988 to 1994) demonstrated that for Caucasian adolescents 12 to 19 years of age, seroprevalence rose from 0.96% to 4.5%. Among 399 adolescents attending an urban adolescent medicine clinic or Job Corps site, the majority of whom were sexually experienced, 12% had antibody to HSV-2, reflecting prior infection. Since 1975, rates of infection with Neisseria gonorrhoeae have declined for all age groups, including teenagers. During the 5-year period of 1992 through 1997, rates of gonorrhea fell 47% among 10- to 14-year-olds and 39% among 15- to 19-year-olds. Despite these trends, more cases of gonorrhea are reported among 15- to 19-year-olds than any other age group. Infection is found in 2% to 10% of adolescent females receiving routine gynecologic care in urban adolescent medicine clinics and in 1% to 5% of asymptomatic males examined in similar settings. Rates of syphilis also have declined for all age groups since 1990. In 1997, rates of primary and secondary syphilis were 0.2/100,000 for 10- to 14year-olds and 4.2/100,000 for 15- to 19-year-olds. References:

Coles FB, Hipp SS. Syphilis among adolescents: the hidden epidemic. Contemp Pediatr. 1996;13:47-62 Division of STD Prevention. Sexually Transmitted Disease Surveillance, 1997. US Department of Health and Human Services, Public Health Service. Atlanta, Ga: Centers for Disease Control and Prevention (CDC); September 1998 Ford CA, Moscicki AB. Control of sexually transmitted diseases in adolescents: the clinician's role. Adv Pediatr Infect Dis. 1995;10:263-305 Krowchuk DP. Sexually transmitted diseases in adolescents: an update. Compr Ther. 1999;25:265-271 Critique 81. Preferred Response: B

[View Question] The development of secondary sexual characteristics occurs in a specific pattern in both males and females during adolescence. Previously known as Tanner stages, these distinct periods of development now are called Sexual Maturity Rating (SMR) stages and are divided into five stages based on pubic hair and breast development in females and pubic hair, testicular, and penile development in males. In both males and females, SMR 1 is the prepubertal stage. It is characterized by the absence of pubic hair in both genders. In the male, the size of the phallus is childlike, with testicular volume less than 1.5 mL. In the female, there is no glandular breast tissue. SMR 2 marks the beginning of puberty. In the male, the testes increase in size and volume, and the scrotum becomes reddened, thinner, and more textured. Although there is no change in the size of the phallus, a small amount of long, downy hair begins to appear along the base of the scrotum and phallus. In females, this pubic hair extends along the labia majora. As described for the girl in the vignette, small breast buds (distinct glandular tissue) can be palpated below the areola in females during SMR 2.

The difference between SMR 2 and SMR 3 in males is an increase in penile length as well as further increases in testicular size. In both males and females, the pubic hair becomes coarse, more curly, and pigmented. SMR 3 breast tissue in females extends beyond the areolar perimeter, with no protrusion of the papilla or nipple. SMR 4 male genital development resembles an adult male, as the length and circumference of the phallus increases. Pubic hair in both sexes becomes denser and curled, but it is less abundant than in the adult. Breasts continue to enlarge at SMR 4 in females. Additionally, the areola and papilla now form a secondary mound that projects from the breast contour. SMR 5 pubic hair extends to the medial surface of the thighs in both males and females and is adult-like in quantity and coarseness. The male phallus and scrotum also have

reached adult size. Although ultimate breast size is variable in females, there is no longer a separate projection of the areola from the remainder of the breast. References: Kreipe RE. Normal somatic adolescent growth and development. In: McAnarney ER, Kreipe RE, Orr DP, Comerci GD, eds. Textbook of Adolescent Medicine. Philadelphia, Pa: WB Saunders Co; 1992:44-67 Kulin HE, Muller J. The biological aspects of puberty. Pediatr Rev. 1996;17:75-86 Neinstein LS, Kaufman FR. Normal physical growth and development. In: Neinstein LS, ed. Adolescent Health Care: A Practical Guide. 3rd ed. Baltimore, Md: Williams & Wilkins; 1996:3-39 Critique 96. Preferred Response: E

[View Question] Leukorrhea is a physiologic vaginal discharge that usually precedes menstruation by 3 to 6 months. It consists of a white, odorless, mucoid discharge that may continue for several years after menarche. Normal saline preparation of the fluid reveals sheets of vaginal epithelial cells without evidence of inflammation, as described for the patient in the vignette. The vaginal pH is usually less than 4.5. Gram stain of the discharge may demonstrate rod-shaped organisms (lactobacilli). The management of physiologic vaginal discharge primarily consists of reassuring the patient and her parents that this type of discharge is normal for her age. An explanation of the cyclic variation of vaginal secretions as regular menstrual cycles develop also may be helpful. The use of sitz baths, cotton undergarments, and a thin panty shield may be suggested if the adolescent is bothered by the discharge. It is important to discourage the use of daily tampons because this may lead to vaginal ulcers. Reassessment in 2 to 3 months may be helpful to confirm the diagnosis and alleviate both patient and parent anxiety. Because leukorrhea is not indicative of an infection, treatment with oral antibiotics is not warranted. Oral azithromycin and cefixime are indicated for cervicitis caused by Chlamydia trachomatis and Neisseria gonorrhoeae, respectively. An antifungal agent, such as clotrimazole vaginal cream, is indicated for the treatment of candidal vaginitis, which is characterized by the appearance of a thick, white discharge. Oral metronidazole is the treatment of choice for trichomonal vaginitis. References: Emans SJ. Vulvovaginal complaints in the adolescent. In: Emans SJH, Laufer MR, Goldstein DP, eds. Pediatric and Adolescent Gynecology. 4th ed. Philadelphia, Pa: Lippincott-Raven; 1998:423-456 Neinstein LS, Himebaugh KS. Vaginitis and cervicitis. In: Neinstein LS, ed. Adolescent Health Care: A Practical Guide. 3rd ed. Baltimore, Md: Williams & Wilkins; 1996:804-820

Critique 113.

Preferred Response: E

[View Question] Precocious puberty is a prime example of the impact that relatively early puberty can have on final adult height. In true precocious puberty, the development of secondary sexual characteristics results from premature reactivation of the hypothalamic gonadotropinreleasing hormone (GnRH) pulse generator. Pulsatile release of luteinizing hormone has a pubertal pattern in children who have true or complete precocious puberty. In patients who have incomplete precocious puberty, extrapituitary secretion of gonadotropins or secretion of gonadal steroids is independent of pulsatile GnRH stimulation. Regardless of the etiology, the increase in gonadal steroid secretion in all forms of sexual precocity leads to premature pubertal changes. These include an increase in height velocity, somatic development, and skeletal maturity. Increased gonadal steroid secretion leads to tall stature in childhood, but relatively short stature as an adult because of premature epiphyseal fusion. Accordingly, the child described in the vignette most likely will be a short adult. Although recent studies of GnRH agonist treatment in children who have precocious puberty have shown promising results, the final adult height in most patients remains below the mean. Final adult height is not determined genetically in patients who have precocious puberty because this condition has a greater influence on final adult height than genetic potential. Administration of growth hormone is not indicated at any time in patients who have precocious puberty. Although radiographs of the hand and wrist can help evaluate skeletal maturity, they do not help in predicting final adult height in patients who have precocious puberty. Standard tables that use bone age to predict final adult height are not valid in this clinical situation. Many children who have early puberty experience normal social development for their chronologic age. However, because of their advanced physical appearance, they may have trouble relating to peers, which could have a deleterious effect on immediate psychosocial development. References: Neinstein LS, Kaufman FR. Abnormal growth and development. In: Neinstein LS, ed. Adolescent Health Care: A Practical Guide. 3rd ed. Baltimore, Md: Williams & Wilkins; 1996:165-193 Styne DM. New aspects in the diagnosis and treatment of pubertal disorders. Pediatr Clin North Am. 1997;44:505-529 Critique 129. Preferred Response: B

[View Question]

A radiograph of the hand and wrist is used to determine skeletal maturity (bone age). This diagnostic study is essential in the evaluation of children and adolescents who have short stature. Because specific bones of the hand and wrist are ossified at certain ages, a radiologist can determine an individuals skeletal maturity with such radiographs. The skeletal maturity or bone age then is compared to the patients chronologic age. Patients who have chronic diseases, such as hypothyroidism, renal failure, and cystic fibrosis, often have a delayed bone age. More commonly, though, a bone age is delayed due to constitutional delay of growth and adolescence. In affected patients, the bone age always is less than the chronologic age. A radiograph of the hand and wrist cannot determine pubertal stage or predict adult height. Pubertal stage is based on the physical development of breasts and pubic hair in girls and pubic hair and genitalia in boys. Adult height can be estimated using midparental height curves and specific formulas that consider the heights of both parents. A bone age should not be used as a screen for chronic illness. Although radiographic findings are abnormal in many chronic conditions, a radiograph of the hand and wrist is too nonspecific to be a useful screening tool. The only method of verifying that puberty has begun is to perform a complete physical examination. References: Neinstein LS, Kaufman FR. Abnormal growth and development. In: Neinstein LS, ed. Adolescent Health Care: A Practical Guide. 3rd ed. Baltimore, Md: Williams & Wilkins; 1996:165-193 Vogiatzi MG, Copeland KC. The short child. Pediatr Rev. 1998;19:92-99 Critique 144. Preferred Response: D

[View Question] The majority of adolescents who exhibit pubertal delay and short stature have constitutional delay of growth and adolescence (CDGA). It is characterized by a normal or near-normal rate of linear growth below but parallel to the 5th percentile during the prepubertal years, delayed skeletal and sexual maturation during adolescence, and a normal final adult height. Other causes of short stature with pubertal delay must be excluded before a definitive diagnosis of CDGA can be made. A complete history, which includes a detailed review of systems, and a careful physical examination are imperative. The typical adolescent who has CDGA is an otherwise healthy boy who has no phenotypic abnormalities and completely normal findings on physical examination. There is often a family history of constitutional delay, with the father commonly being short as a child and experiencing a late pubertal growth spurt. Some clinicians use the term late bloomer to describe this pattern of growth.

The preliminary laboratory evaluation for delayed puberty should include a complete blood count, urinalysis, erythrocyte sedimentation rate, thyroid function tests, and bone age. Determination of the bone age for skeletal maturity is an essential part of the evaluation because skeletal maturity often is delayed in adolescents who have chronic illness, hypopituitarism, and hypothyroidism. In addition, the bone age can be compared to the patients height age by locating the corresponding age on a growth curve at which the patients height would be equal to the 50th percentile. In CDGA, the bone age always is delayed, usually is equal to the height age, and is less than the chronologic age. References: Neinstein LS, Kaufman FR. Abnormal growth and development. In: Neinstein LS, ed. Adolescent Health Care: A Practical Guide. 3rd ed. Baltimore, Md: Williams & Wilkins; 1996:165-193 Vogiatzi MG, Copeland KC. The short child. Pediatr Rev. 1998;19:92-99 Critique 161. Preferred Response: B

[View Question] Dysfunctional uterine bleeding (DUB) is defined as irregular, painless, prolonged endometrial bleeding. This condition can be accompanied by a wide range of symptoms, depending on the amount of blood loss. In the young adolescent it is usually due to anovulatory cycles. The anovulatory cycles are the result of an immature hypothalamic-pituitary-ovarian axis. Regular ovulatory cycles do not develop for 1 to 2 years after menarche, and approximately 10% to 20% of cycles remain anovulatory for up to 5 years after menarche. Because DUB is essentially a diagnosis of exclusion, a careful history and physical examination are required to rule out other conditions that can cause abnormal vaginal bleeding in the adolescent. In sexually active adolescents, pregnancy and pregnancy-related complications as well as sexually transmitted diseases such as pelvic inflammatory disease always must be considered, as should trauma from forceful sexual intercourse. Occasionally, a blood dyscrasia or coagulation defect will present as abnormal vaginal bleeding. However, patients who have these disorders usually are severely anemic and have a history of consistently long and heavy periods. Finally, thyroid disease can be the etiology of excessive vaginal bleeding. The management of an adolescent who has DUB depends on the degree of anemia and whether the patient is hemodynamically stable. Patients who have a normal hematocrit and normal findings on physical and pelvic examinations can be managed conservatively with a menstrual calendar and reassurance. However, any patient who has anemia should be treated with oral ferrous sulfate and combined hormonal therapy (estrogen and progesterone) or progesterone alone. Outpatient administration of combined oral contraceptives is convenient for the adolescent who has stable vital signs. However, the patient who has severe anemia (hemoglobin (<70 to 80 g/L [<7 to 8 mg/dL]) and hemodynamic instability should be managed in the hospital with intravenous conjugated estrogen, antiemetics, and, if indicated, a blood transfusion.

The adolescent described in the vignette has normal vital signs and mild anemia. Administration of oral ferrous sulfate should be the first step in management, with combined hormonal therapy also considered. Oral estrogen supplementation alone is not appropriate therapy; progesterone also is needed to stabilize the endometrium. A serum von Willebrand panel is indicated in patients who are severely anemic at initial presentation or who have a history of persistent excessive bleeding. Pelvic ultrasonography should be reserved for those in whom a complication of pregnancy is suspected or to clarify pelvic anatomy. Referral to a gynecologist for further evaluation is not warranted in a patient who is stable. References: Bravender T, Emans SJ. Menstrual disorders. Dysfunctional uterine bleeding. Pediatr Clin North Am. 1999;46:545-553 Braverman PK, Soundheimer SJ. Menstrual disorders. Pediatr Rev. 1997;18:17-26 Iglesias EA, Coupey SM. Menstrual cycle abnormalities: diagnosis and management. Adolesc Med. 1999;10:255-273 Critique 177. Preferred Response: E

[View Question] Sexual development in a boy begins at approximately 11.5 years of age, with a range of 9.5 to 13.5 years. In 98% of males, the first physical sign of puberty is testicular enlargement. This is followed by an increase in the size of the phallus, epididymis, and prostate and the appearance of pubic hair. The typical sequence of events is adrenarche, beginning of growth spurt, testicular development, beginning of pubic hair, and peak height velocity. On the average, puberty is complete in 3 years, but it can range in duration from 2 to 5 years. The adolescent boy described in the vignette has completed Sexual Maturity Rating (SMR) (Tanner) stage 2. His testes and penis have enlarged slightly. In addition, the distribution and quality of his pubic hair are consistent with SMR stage 2. Although his height is only at the 5th percentile, he has not yet experienced his peak height velocity. As noted previously, this is the last pubertal event to occur in males. Accordingly, it is most appropriate to reassure him that his short stature is due to his early stage of pubertal development and to re-evaluate him in 6 months. At that time, a complete physical examination should be performed to confirm the progression of pubertal development. The pubertal development of the boy described in the vignette is following the typical sequence of pubertal events. Medications, including growth hormone or androgens, are inappropriate. Oral thyroxine therapy should be reserved for patients who exhibit signs and symptoms of hypothyroidism that is confirmed by laboratory evaluations. Although radiography of the hand and wrist will help determine skeletal maturity, it is not necessary in this patient, who is only at SMR stage 2. References:

Kreipe RE. Normal somatic adolescent growth and development. In: McAnarney ER, Kreipe RE, Orr DP, Comerci GD, eds. Textbook of Adolescent Medicine. Philadelphia, Pa: WB Saunders Co; 1992:44-67 Kulin HE, Muller J. The biological aspects of puberty. Pediatr Rev. 1996;17:75-86 Neinstein LS, Kaufman FR. Normal physical growth and development. In: Neinstein LS, ed. Adolescent Health Care: A Practical Guide. 3rd ed. Baltimore, Md: Williams & Wilkins; 1996:3-39 Critique 193. Preferred Response: E

[View Question] Physiologic leukorrhea is a normal vaginal discharge that typically precedes menstruation by 3 to 6 months. It is usually white, odorless, and mucoid in appearance and may continue for several years following menarche. Normal saline preparation or wet mount of the fluid reveals sheets of vaginal epithelial cells without evidence of significant inflammation, as described in the vignette. It is helpful to obtain information regarding pubertal development when assessing a young adolescent who has a vaginal discharge because leukorrhea occurs at a specific time prior to menarche. The clinician can estimate when menarche is likely to occur, based on the onset of thelarche and peak height velocity. Breast development usually begins 2 years before the onset of menstruation, and the growth spurt occurs 1 to 1.5 years before menses. In addition, approximately two thirds of females menstruate when they reach Sexual Maturity Rating (SMR) (Tanner) stage 3. The young girl described in the vignette is premenarcheal, but based on the information provided, she most likely will begin her menses soon because it has been 2 years since thelarche and she already has reached SMR stage 3. Findings on the microscopic evaluation of the vaginal discharge are consistent with a diagnosis of physiologic leukorrhea. Significant vulvar pruritus is associated with Candida albicans vaginitis. Additionally, the discharge associated with this has a cottage cheese or curdy appearance. In bacterial vaginosis, a fishy, amine odor is created when 10% potassium hydroxide is combined with a specimen of vaginal fluid. Chlamydia and herpes simplex infections often are accompanied by dysuria, but they rarely cause significant vaginal discharge. References: Emans SJ. Vulvovaginal complaints in the adolescent. In: Emans JSH, Laufer MR, Goldstein DP, eds. Pediatric and Adolescent Gynecology. 4th ed. Philadelphia, Pa: Lippincott-Raven; 1998:423-456 Neinstein LS, Himebaugh KS. Vaginitis and cervicitis. In: Neinstein LS, ed. Adolescent Health Care: A Practical Guide. 3rd ed. Baltimore, Md: Williams & Wilkins; 1996:804-820 Critique 208. Preferred Response: B

[View Question] The normal ranges for hemoglobin vary substantially with age and gender. Failure to recognize these variations will result in many children being classified inappropriately as either normal or anemic. Tables (Figure 208A) of normal values are readily available in pediatric textbooks and handbooks. Hemoglobin levels are highest at birth, when the lower limit of normal is 130 g/L (13 g/dL). It falls to a lower limit of 90 to 94 g/L (9 to 9.4 g/dL) at 2 to 3 months of age, then slowly increases, reaching 110 g/L (11.0 g/dL) at 6 months, which is the lower limit of normal for this age. The lower limit of normal hemoglobin levels is approximately 120 g/L (12 g/dL) at 11 years of age for both boys and girls. At this point, girls have attained the lower limit of normal of adult females. In contrast, the hormonal effect of increased androgens as boys enter puberty causes hemoglobin levels to rise steadily until approximately 18 years of age, when the adult lower limit of 135 g/L (13.5 g/dL) is attained. Substantial evidence suggests that the normal range of hemoglobin averages about 50 g/L (0.5 g/dL) lower in African-Americans compared with Caucasians. Therefore, overdiagnosis and overtreatment of mild anemia is more likely in this population. Precise use of normal values is important because errors of 50 g/L (0.5 g/dL) in the normal range can allow misclassification of 10% to 15% of children who have normal levels. The result can be a large number of children in whom either anemia is missed or unnecessary studies and treatments are undertaken. References: Brugnara C. Appendices: Reference values in infancy and childhood. In: Nathan DG, Orkin SH, eds. Nathan and Oski's Hematology of Infancy and Childhood. 5th ed. Philadelphia, Pa: WB Saunders Co; 1998:i-xxviii Ebel BE, Raffini L. Hematology. In: Siberry GK, Iannone R, eds. The Harriet Lane Handbook. 15th ed. St Louis, Mo: Mosby-Year Book, Inc; 2000:323-325 Critique 225. Preferred Response: D

[View Question] A vaginal discharge in a prepubertal girl often is nonspecific and caused by poor hygiene. In contrast, vaginitis in the adolescent usually has a specific cause and frequently is related to sexual activity. To distinguish between the two types of discharge, specific aspects of the history and physical examination must be reviewed. The clinician must inquire about a history of sexual molestation or activity in all girls who have a vaginal discharge, regardless of their pubertal status. In addition, documenting a recent history of oral broad-spectrum antibiotic use is important because such a regimen may lead to an infection with Candida albicans. Additional important information includes a history of symptoms (odor, pruritus, quantity), the use of other medications such as oral contraceptives, similar episodes of vulvovaginitis in the past, and previous treatments.

The physical examination should include an inspection of the external genitalia in both virginal and sexually active girls. A speculum examination usually is not necessary in the virginal adolescent; samples of the vaginal fluid can be obtained with a saline-moistened, cotton-tipped applicator gently inserted through the hymenal opening. Both gross examination of vaginal secretions and microscopic examination of the fluid will assist with the diagnosis. A thin, white, mucoid discharge in a premenarcheal girl, as described in the vignette, is consistent with physiologic leukorrhea. A saline preparation slide that reveals sheets of vaginal epithelial cells with minimum inflammation confirms the diagnosis. In addition, the vaginal pH will be less than 4.5. Although Candida vaginitis also may have a normal pH, the discharge is typically thick and curdy. Pseudohyphae can be seen on the potassium hydroxide preparation. The discharge associated with trichomoniasis typically is white or yellow, bubbly, and frothy. Motile flagellated organisms are seen on saline preparation along with large numbers of leukocytes. The vaginal pH is elevated above 4.5 both in patients who have Trichomonas vaginitis and those who have bacterial vaginosis. Symptomatic infections with Neisseria gonorrhoeae may be accompanied by a greenish-yellow discharge from the vagina. References: Emans SJ. Vulvovaginal complaints in the adolescent. In: Emans SJH, Laufer MR, Goldstein DP, eds. Pediatric and Adolescent Gynecology. 4th ed. Philadelphia, Pa: Lippincott-Raven; 1998:423-456 Neinstein LS, Himebaugh KS. Vaginitis and cervicitis. In: Neinstein LS, ed. Adolescent Health Care: A Practical Guide. 3rd ed. Baltimore, Md: Williams & Wilkins; 1996:804-820 Critique 240. Preferred Response: C

[View Question] Dysfunctional uterine bleeding (DUB) is defined as irregular, painless endometrial bleeding that is prolonged, excessive, and unpatterned. It is associated most commonly with anovulatory menstrual cycles in the young adolescent female due to immaturity of the hypothalamic-pituitary-ovarian axis. Generally, regular ovulatory cycles do not develop until 1 to 2 years after menarche. When anovulatory cycles occur, the endometrium experiences persistent estrogen stimulation that is unopposed by progesterone. It grows abnormally and eventually begins to break down. Incomplete sloughing of the endometrium leads to irregular, prolonged bleeding. After a thorough history is obtained and physical examination is performed, the laboratory evaluation of irregular vaginal bleeding should include a pregnancy test and a complete blood count to assess for anemia. The degree of anemia will help guide the treatment plan.

The goals of treatment in DUB are threefold: to correct hemodynamic instability, to control heavy bleeding, and to correct anemia. An additional goal is to prevent future episodes. In most adolescents, these goals can be achieved by hormonal treatment with oral contraceptive pills (OCPs). Severe anemia (hemoglobin 70 to 80 g/L [<7 to 8 mg/dL]) or hemodynamic instability should be treated in the hospital with intravenous conjugated estrogen and, in some cases, whole blood transfusion. Most patients, such as the girl described in the vignette, are stable and can be treated on an outpatient basis with OCPs and iron therapy. The duration of therapy depends upon the degree of anemia at initial presentation. Dilatation and curettage rarely is necessary in adolescents who experience prolonged uterine bleeding; it is used only as a last resort. Because DUB can be associated with hemodynamic instability and anemia, reassuring the patient that her menstrual cycles are normal is inappropriate. References: Bravender T, Emans SJ. Menstrual disorders. Dysfunctional uterine bleeding. Pediatr Clin North Am. 1999;46:545-553 Braverman PK, Soundheimer SJ. Menstrual disorders. Pediatr Rev. 1997;18:17-26 Iglesias EA, Coupey SM. Menstrual cycle abnormalities: diagnosis and management. Adolesc Med. 1999;10:255-273 Critique 255. Preferred Response: E

[View Question] There is great variation in the timing of puberty and in the attainment of physical maturity among adolescents. Although the sequence of pubertal events takes place in an orderly fashion, the age at which secondary sexual characteristics appear varies and is primarily genetically determined. Specific events that occur as a part of puberty, however, such as peak height velocity (PHV) and menarche, are correlated more closely with the Sexual Maturity Rating (SMR) (Tanner) stage than with chronologic age. The average time for the completion of puberty is 3 to 4 years, but it can range from 2 to 5 years in males and 1.5 to 8 years in females. As puberty progresses, adolescents become increasingly concerned with the physical changes that are occurring and often compare the appearance of their bodies with that of their peers. Anxiety regarding height is common, especially among boys, who worry that that they always will be the shortest person in their group. The clinician usually can alleviate this anxiety after performing a thorough history and physical examination. Determining the SMR stage often can aid the clinician who is counseling the adolescent. If the SMR stage is less than 5, the adolescent can be reassured that his final adult height has not yet been attained. Despite his chronologic age, the adolescent boy described in the vignette is early in puberty, and he is otherwise healthy. He should not experience his PHV until after he attains SMR stage 3 pubic hair and stage 4 genitalia. Depending on when his father completed

puberty, the time of PHV in this boy may even be later. Accordingly, reassurance and reevaluation in 4 to 6 months is the most appropriate management. The administration of growth hormone or testosterone is not indicated for this adolescent because neither puberty nor growth is delayed. Further, his height, which has been following the 10th percentile, is not consistent with growth hormone deficiency or thyroid disease. A magnetic resonance imaging scan of the head in an otherwise healthy, asymptomatic patient is not warranted. References: Kreipe RE. Normal somatic adolescent growth and development. In: McAnarney ER, Kreipe RE, Orr DP, Comerci GD, eds. Textbook of Adolescent Medicine. Philadelphia, Pa: WB Saunders Co; 1992:44-67 Kulin HE, Muller J. The biological aspects of puberty. Pediatr Rev. 1996;17:75-86 Neinstein LS, Kaufman FR. Normal physical growth and development. In: Neinstein LS, ed. Adolescent Health Care: A Practical Guide. 3rd ed. Baltimore, Md: Williams & Wilkins; 1996:3-39 2000 Self-Assessment Exercise XXIV. Adolescent medicine and gynecology [Return to Category List] Questions [Print Directions] Question 10. Answer.

The mother of a 8-year-old girl asks you when she should expect her child to begin puberty. You respond that it usually occurs within a standardized age range among boys and girls in the United States. Of the following, the BEST estimate for the age range (in years) during which puberty occurs is: A. B. C. D. E. 7 to 11 for boys and 10 to 14 for girls 9 to 13 for both boys and girls 11 to 15 for boys and 7 to 11 for girls 13 to 17 for boys and 11 to 15 for girls 15 to 19 for boys and 13 to 17 for girls Answer.

Question 36.

A 16-year-old girl is concerned that she has not started her menses. Both her mother and sister menstruated at age 13. She is receiving no medications and denies strenuous exercise

or dieting. Her height and weight are at the 75th percentile for age. She has sexual maturity rating (Tanner) stage 4 breast and pubic hair development. Ultrasonography of the pelvis reveals normal female anatomy. Of the following, the MOST likely diagnosis is A. B. C. D. E. imperforate hymen physiologic pubertal delay prolactinoma testicular feminization syndrome Turner syndrome Answer.

Question 65.

A 13-year-old boy is seen in your office because of breast enlargement over the past 6 months. He denies pain, galactorrhea, and marijuana use. He is taking no medications. Physical examination reveals sexual maturity rating (Tanner) stage 2 genitalia and pubic hair growth and asymmetric breast buds beneath each nipple, with the left measuring 1 cm and the right 3 cm. Of the following, the most appropriate INITIAL management for this boy is A. B. C. D. E. computed tomography of the head with enhancement magnetic resonance imaging of the abdomen serum beta-human chorionic gonadotropin level reassurance that this is a normal occurrence ultrasonography of the testes Answer.

Question 99.

A 16-year-old girl has had a purulent vaginal discharge for the past 2 weeks without other symptoms. Her last menstrual period was 3 weeks ago. Pelvic examination reveals a friable cervix with purulent material coming from the os. There is no cervical or adnexal tenderness to palpation. Of the following, in addition to oral azithromycin, the most appropriate treatment is A. B. C. D. acyclovir cefazolin cefixime doxycycline

E.

metronidazole

Question 129. Answer. You are evaluating an otherwise healthy 15-year-old boy who has short stature. Radiography of his hand and wrist reveals a bone age of 13.5 years. Of the following, the MOST likely explanation for the disparity between the chronologic and bone age is A. B. C. D. E. chronic illness growth hormone deficiency hypothyroidism normal adolescent variant nutritional deficiency

Question 165. Answer. A 14-year-old boy has no known underlying chronic medical conditions. Results of his physical examination are normal. His height is 165 cm (65 in), which is at the 50th percentile for age, and he has a sexual maturity rating of 2 for pubic hair and genitalia. His mother is 168 cm (66 in) tall and his father is 177 cm (70 in) tall. His parents are interested in knowing what the boy's ultimate height will be. Among the following, the statement that MOST accurately describes this boy's predicted stature is that A. B. C. he probably will be shorter than his father he probably will be taller than his father he probably will be the same height as his father

D. his ultimate height cannot be predicted accurately because of his delayed pubertal development E. his ultimate height cannot be predicted accurately without obtaining a bone age

Question 197. Answer. A 15-year-old girl complains of a malodorous, non-bloody vaginal discharge for 1 week. There is no vulvar itching or discomfort. Her only medication is oral contraceptives. A thin, white, homogeneous vaginal discharge is noted on pelvic examination. The vaginal fluid has a pH of 5 and a "fishy" odor when mixed with 10% potassium hydroxide. A microscopic examination of the vaginal fluid MOST likely would show A. clue cells

B. C. D. E.

lactobacilli leukocytes motile trichomonads pseudohyphae

Question 231. Answer. You are a guest speaker at a local high school health class. A 15-year-old girl asks if you can predict her adult height. Of the following, the MOST appropriate response is that A. B. C. D. E. adult height cannot be predicted adult height depends more on paternal than maternal height age at onset of puberty predicts height amount of sleep during puberty affects height girls' heights rarely increase more than 5.08 cm (2 in) after menarche

Question 256. Answer. During a routine health supervision visit, a 16-year-old girl complains of lower abdominal pain during the first 2 days of her menses. It is accompanied by bloating and vomiting and occasionally causes her to miss school and sports. Menarche occurred at age 13. Her menstrual cycle is regular, lasting 4 to 5 days, with moderate flow. Of the following, the MOST appropriate management of these symptoms is A. B. C. D. E. administration of acetaminophen administration of a prostaglandin inhibitor dietary modification increased exercise reassurance that pain with menses is normal

Answers Critique 10 Preferred Response: B

[View Question] Puberty occurs primarily between 9 and 13 years of age for both boys and girls in the United States. It is considered to be premature if it occurs before age 9 in a boy or age 8 in a girl and to be delayed in girls if breast development has not begun by age 13.

The timing of somatic changes from the onset of puberty to its completion varies widely among individuals, possibly due to differing ethnic, environmental, and genetic influences. Most adolescents experience pubertal events in an orderly sequence, but the onset and time interval within the sequence of events is quite variable. Pubertal events correlate more closely with Tanner stage or sexual maturity rating than with chronologic age. The first sign of pubertal development in most girls is the onset of breast budding, which occurs on the average at 11.2 years of age, but can appear at any time within the age range of 9 to 13 years. The average length of time for completion of puberty in a girl is 4 years (range, 1.5 to 8 years). For the average adolescent, menarche follows 2 to 2.5 years after the onset of breast development and 3.3 years after the growth spurt. In approximately 98% of boys, pubertal development begins with testicular enlargement. Thinning and reddening of the scrotum also occurs at this time. The mean age for the initiation of puberty is 11.5 years (range, 9.5 to 13.5 years). The average length of time for completion of puberty is 3 years for boys, but it can range from 2 to 5 years. In relationship to sexual development, peak height velocity occurs later in males than in females. References: Joffe A. Adolescent medicine. In: Oski FA, DeAngelis CD, Feigin RD, McMillan JA, Warshaw JB, eds. Principles and Practice of Pediatrics. 2nd ed. Philadelphia, Pa: JB Lippincott Co; 1994:763-775 Kreipe RE. Normal somatic adolescent growth and development. In: McAnarney ER, Kreipe RE, Orr DP, Comerci GD, eds. Textbook of Adolescent Medicine. Philadelphia, Pa: WB Saunders Co; 1992:44-67 Kulin HE, Muller J. The biological aspects of puberty. Pediatr Rev. 1996;17:75-86 Neinstein LS, Kaufman FR. Normal physical growth and development. In: Neinstein LS, ed. Adolescent Health Care: A Practical Guide. 3rd ed. Baltimore, Md: Williams & Wilkins; 1996:22-24 Critique 36 Preferred Response: B

[View Question] The adolescent described in the vignette has primary amenorrhea, a term that traditionally has been used to characterize a female who never has started her menses. Clinically, primary amenorrhea is the lack of menses by age 16 years or 2 years after reaching sexual maturation. However, because pubertal development and menses can begin at such variable times in young females, these definitions are only general guidelines; each adolescent should be evaluated individually for concern regarding menses or other aspects of pubertal development.

In the United States, the median age of menarche is approximately 12.5 years, with a range from 9 to 16 years. Most girls begin their menses 2 to 2.5 years after thelarche (the onset of breast development) and 3.3 years following the growth spurt. Further, 60% of females will have menstruated by the time they reach sexual maturity rating (Tanner) stage 4, and 95% within 1 year of reaching sexual maturity rating stage 5. Accordingly, a lack of menses 3 years after beginning puberty, especially in a female who has reached sexual maturity rating stage 4 or 5, such as the girl described in the vignette, warrants evaluation. The differential diagnosis of an adolescent who has primary amenorrhea includes hypothalamic/pituitary disorders, ovarian dysfunction, adrenal abnormalities, and anatomic variants of the genital tract. Some authors have divided amenorrhea into four categories: 1) amenorrhea with pubertal delay, 2) amenorrhea with normal pubertal development, 3) genital tract anomalies, and 4) hyperandrogenic anovulation or polycystic ovary syndrome. Regardless of the algorithm used, the approach to primary amenorrhea should include consideration of each of these entities. The age of the adolescent described in the vignette is consistent with primary amenorrhea, and her sexual maturation is progressing appropriately. It is likely that she has physiologic pubertal delay because she has not yet completed pubertal development, and ultrasonography of the pelvis is normal. The results of ultrasonography eliminate the diagnosis of an imperforate hymen because there is no evidence of outflow tract obstruction. Patients who have imperforate hymen also may present with cyclic abdominal pain or a palpable midline mass. Patients who have testicular feminization have normal breast development but lack pubic or axillary hair. Further, pelvic ultrasonography reveals agenesis of the vagina and uterus. Individuals who have Turner syndrome have no pubertal development as a result of the absence of one of the X chromosomes and ovarian failure associated with streaked gonads. The presence of a prolactinoma might cause headaches and galactorrhea, although this is not universal. References: Braverman PK, Sondheimer SJ. Menstrual disorders. Pediatr Rev. 1997;18:17-25 Neinstein LS. Amenorrhea. In: Adolescent Health Care: A Practical Guide. 3rd ed. Baltimore, Md: Williams & Wilkins; 1996:783-795 O'Connell BJ. The pediatrician and the sexually active adolescent. Treatment of common menstrual disorders. Pediatr Clin North Am. 1997;44:1391-1404 Prose CC, Ford CA, Lovely LP. Evaluating amenorrhea: the pediatrician's role. Contemporary Pediatrics. 1998;15: 83-110

Critique 65

Preferred Response: D

[View Question] Gynecomastia commonly occurs in boys progressing through puberty. An estimated 70% of adolescent males are said to be affected, with a peak prevalence occurring between ages 12 and 15. In general, this corresponds to sexual maturity rating (Tanner) stage 2 to 3 in the young male. Like breast development in the pubertal female, transient pubertal gynecomastia may be asymmetric, although concurrent or sequential involvement of both breasts can occur. In Type I gynecomastia, the breast tissue is localized under the areola; in Type II, it extends beyond the areolar perimeter. In most cases, the breast tissue is 2 to 3 cm in size, firm, and tender. Galactorrhea rarely is present and may indicate self-stimulation or illicit drug use, especially marijuana. Transient pubertal gynecomastia is believed to result from a temporary imbalance between circulating estrogens and androgens. Alterations in the ratio of these hormones can lead to an increase in estrogen relative to testosterone. Because the typical case of gynecomastia begins in early puberty, nonphysiologic causes must be considered in the older male who has completed puberty (sexual maturity rating [Tanner] stage 5) or if the condition persists for more than 2 years. In addition, the onset of breast development in a prepubertal boy warrants a more thorough evaluation. Pathologic causes of gynecomastia include liver and adrenal tumors, testicular neoplasms, thyroid disease, malnutrition, hypogonadism, and Klinefelter syndrome (47XXY). Several drugs also are associated with gynecomastia, including phenothiazines, diazepam, digoxin, spironolactone, and cimetidine. Substances of abuse (marijuana, alcohol, amphetamines, heroin) also have been implicated as causes of gynecomastia. The boy described in the vignette most likely has transient pubertal gynecomastia. There is no need for diagnostic testing (eg, magnetic resonance imaging, computed tomography, ultrasonography, or laboratory tests). The primary treatment for transient pubertal gynecomastia is reassurance that the condition is self-limited and should resolve within 1 to 2 years. The adolescent also should be told explicitly that he is not turning into a girl. In most cases, he should be re-examined periodically until resolution occurs. Plastic surgery may be considered for persistent breast enlargement or moderate-to-severe gynecomastia with psychological difficulties. References: Beach RK. Breast disorders. In: McAnarney ER, Kreipe RE, Orr DP, Comerci GD, eds. Textbook of Adolescent Medicine. Philadelphia, Pa: WB Saunders Co; 1992: 720-728 Neinstein LS. Gynecomastia. Adolescent Health Care: A Practical Guide. 3rd ed. Baltimore, Md: Williams & Wilkins; 1996:210-214 Wilson MD. Breast problems. In: Oski FA, DeAngelis CD, Feigin RD, McMillan JA, Warshaw JB, eds. Principles and Practice of Pediatrics. 2nd ed. Philadelphia, Pa: JB Lippincott Co; 1994:775-777

Critique 99

Preferred Response: C

[View Question] The presence of a malodorous vaginal discharge in an adolescent usually has a specific cause that often is related to sexual activity. Common etiologies for vaginal discharge in this age group include Trichomonas vaginalis; bacterial vaginosis; Candida; and cervicitis due to Neisseria gonorrhoeae, Chlamydia trachomatis, or herpes simplex. A friable cervix with purulent material coming from the os, as described for the girl in the vignette, is characteristic of endocervicitis. Endocervicitis is an inflammation of the mucous membranes of the endocervix that often is accompanied by polymorphonuclear cells and lymphocytes. Symptoms include vaginal discharge, dyspareunia, postcoital bleeding, or intramenstrual bleeding. Dysuria may be present if there is an associated urethral infection. The presence of abdominal pain should alert the physician to the possibility of pelvic inflammatory disease (PID). Signs of cervicitis include cervical erythema, friability, and mucopurulent discharge. Fever, lower abdominal pain, cervical motion tenderness, adnexal tenderness, or an adnexal mass are seen in the patient who has PID. Treatment of cervicitis should be directed initially at the most likely etiology, based on the findings on physical examination. For the girl described in the vignette, the most appropriate treatment is oral azithromycin and cefixime to treat C trachomatis and N gonorrhoeae. Doxycycline is an alternative therapy for Chlamydia infection, but the patient in the vignette will be treated effectively for this infection with a single oral dose of azithromycin. Alternative single-dose regimens for gonorrhea include intramuscular ceftriaxone, oral ciprofloxacin, and oral ofloxacin. Simultaneous treatment for Chlamydia is recommended when treating gonorrhea because of the frequent coexistence of these two infections. Single-dose therapy is not adequate for the treatment of PID. Acyclovir is the treatment of choice for primary symptomatic herpes simplex infection, which is manifested by cervical and vulvar ulcers. Single-dose metronidazole is the treatment of choice for trichomoniasis, which usually is associated with inflammation of the vaginal mucosa. Cefazolin is not appropriate therapy for cervicitis. References: Gittes EB, Irwin CE Jr. Sexually transmitted diseases in adolescents. Pediatr Rev. 1993;14:180-189 Lappa S, Moscicki AB. The pediatrician and the sexually active adolescent. A primer for sexually transmitted diseases. Pediatr Clin North Am. 1997;44:1405-1445 Vandeven AM, Emans SJ. Vulvovaginitis in the child and adolescent. Pediatr Rev. 1993;14:141-147 Critique 129 Preferred Response: D

[View Question] An adolescent's bone age and chronologic age may be disparate by as much as 2 years, as reported for the boy in the vignette, and still be within normal limits. The two most common reasons for short stature in the adolescent who is otherwise healthy are familial short stature (FSS) and constitutional delay of growth in adolescence (CDGA). Both are normal variations in growth and can be distinguished from each other by radiography of the hand and wrist. In FSS, the bone age usually is not delayed and is consistent with the chronologic age of the patient. In the short adolescent who has CDGA, however, both skeletal and sexual maturation are delayed. The bone age typically is delayed and often corresponds to the height age of the patient. The usual growth pattern in patients who have CDGA is a deceleration of linear growth in the first 2 to 3 years of life, a normal or near-normal rate of growth that is below or parallel to the 5th percentile in the prepubertal years, and a final adult height that usually is within the normal range. Often described as late bloomers, these patients may have a family history of delayed puberty. Several other conditions also can cause short stature, but they usually can be differentiated from FSS and CDGA by the history and physical examination. Growth hormone deficiency, hypothyroidism, and nutritional deficiency all are associated with short stature and a delayed bone age, but they are highly unlikely in an otherwise healthy adolescent who has been following his growth curve. In children who have chronic illness, the bone age may or may not be delayed. References: Cohen P, Rosenfeld RG. Disorders of growth. In: McAnarney ER, Kreipe RE, Orr DP, Comerci GD, eds. Textbook of Adolescent Medicine. Philadelphia, Pa: WB Saunders Company; 1992:494-5038 Rosenthal SM, Wilson DM. Pediatric endocrinology. In: Rudolph AM, Kamei RK, eds. Rudolph's Fundamentals of Pediatrics. Norwalk, Conn: Appleton & Lange; 1994: 583-585 Vogiatzi MG, Copeland KC. The short child. Pediatr Rev. 1998;19:92-99 Critique 165 Preferred Response: B

[View Question] Although general principles can be used to estimate adult height, there is no definitive method that can accurately predict ultimate adult height. Factors that need to be considered in making such predictions include the parents' height and the child's growth pattern, current height, pubertal stage of development, and bone age. Almost 25% of adult height occurs during pubertal maturation, with peak height velocity occurring between sexual maturity rating (SMR) (Tanner) stages 3 and 4. Because girls on average enter puberty several years earlier than boys, most of their linear growth is complete by 14 years of age. In contrast, most boys do not achieve the majority of their

linear growth until after age 16 years. Children who have constitutional growth delay will have a delayed bone age and a delay in the onset of pubertal development. These children have a corresponding delay in epiphyseal closure and achievement of final adult height. Predictions of adult stature in prepubertal children frequently rely on measurement of bone age, but once pubertal development has begun, ultimate growth potential can be correlated best with the child's SMR. Tanner has modified standard growth curves to include curves for early and late developers to help assess the pattern of growth in these children. Zadik developed a computer model for predicting final height in pubertal boys based upon a calculation of the pubertal growth spurt, which obviates the need for determining bone age. Trying to predict the final stature of the boy described in the vignette, who is 165 cm (65 in) and at the 50th percentile at 14 years of age, requires estimation of his remaining growth potential. A boy whose bone age equals his chronologic age of 14 years should be at SMR stage 3 or 4 for genital development and would have several years of growth remaining that will result in his ultimate height being approximately at the 50th percentile for an adult male (70 in or 177 cm). However, the boy described in the vignette is only at SMR stage 2 for genital development, indicating that his sexual development is somewhat delayed compared with his peers. He has constitutional growth delay and would have a delayed bone age. Accordingly, this boy's period of linear growth should extend for 12 to 18 months beyond that seen in a boy who would be at SMR stage 3 or 4 at the same age. This will result in an adult height that is greater than the 50th percentile. Thus, he likely will be taller than his father who is 70 in or 177 cm and at the 50th percentile. Numerous methods are available to predict height in children and adolescents. A common method to predict height (5 cm) uses the following formulas Boys: Father's ht (cm) + Mother's ht (cm) + 13 2 Girls: Father's ht (cm) + Mother's ht (cm) - 13 2 However, if this formula has been used without taking into consideration the delayed sexual development of the boy in the vignette, it would have underestimated his ultimate adult height. Other charts and methods have been developed by Bayley-Pinneau, Tanner, and Roche-Wainer-Thissen. Whichever method is used, clinicians must remember that these predictions are only approximations and advise parents that there is no method to predict their child's ultimate height accurately. References: Biro FM. Physical growth and development. In: Friedman SB, Fisher M, Schonberg SK, Alderman EM, eds. Comprehensive Adolescent Health Care. 2nd ed. St Louis, Mo: MosbyYear Book, Inc; 1998:28-33

Plotnick LP. Growth, growth hormone, pituitary disorders. In: Oski FA, DeAngelis CD, Feigin RD, McMillan JA, Warshaw JB, eds. Principles and Practice of Pediatrics. 2nd ed. Philadelphia, Pa: JB Lippincott Co; 1994: 1973-1981 Rapaport R. Growth curves and adult height estimation for adolescents. Pediatr Rev. 1989;11:70 Zadik Z, Segal N, Limony Y. Final height prediction models for pubertal boys. Acta Paediatr Suppl. 1996;417: 53-56 Critique 197 Preferred Response: A

[View Question] Bacterial vaginosis (BV) occurs in 4% to 15% of college students and 30% to 40% of women attending a sexually transmitted disease clinic. Although it can occur in virginal females, it is much more common among those who are sexually active. Reported risk factors include the presence of an intrauterine device, a prior sexually transmitted disease, smoking, low socioeconomic status, and an uncircumcised partner. As described in the vignette, the predominant symptom is a malodorous vaginal discharge. BV, previously known as Gardnerella or nonspecific vaginitis, results from alteration of normal vaginal flora. Although it is unclear why this occurs, there is an increased concentration of G vaginalis, genital mycoplasmas such as Mycoplasma hominis, and anaerobic bacteria, especially Mobiluncus and Bacteroides sp. In addition to the overgrowth of these bacteria, there is a decrease in the normal number of hydrogen peroxide-producing lactobacilli. This results in an elevated (alkaline) vaginal pH as well as the production of amines, putrescine, and cadaverine, which leads to the typical fishy odor of the discharge. Clinical characteristics of BV include the presence of a homogeneous, malodorous vaginal discharge on pelvic examination that may be white, yellow, or gray. The discharge often adheres to the vaginal walls and has a pH of more than 4.5. The addition of 10% potassium hydroxide to a sample of the vaginal discharge often produces a fishy or amine odor (positive whiff test). The finding of clue cells, which are epithelial cells coated with large numbers of bacteria that obscure the cell borders on a saline preparation of the vaginal fluid also is consistent with BV. Gram stain of the vaginal discharge will show four or fewer lactobacilli per oil immersion field along with other bacteria (Gram-positive and -negative coccobacilli and curved Gram-negative rods). Cultures for G vaginalis are not helpful in making the diagnosis. Three of four criteria are required to make the clinical diagnosis of BV: 1) vaginal discharge, 2) pH 4.5, 3) positive whiff test, and 4) at least 20% clue cells on saline preparation (wet mount). The clinical presentation and results of the physical examination and diagnostic studies described in the vignette suggest a diagnosis of BV. A saline preparation most likely would show clue cells. Large numbers of leukocytes may be seen with trichomoniasis, cervicitis, and occasionally Candida infections; they usually are not seen in BV. Motile trichomonads are diagnostic of a trichomonal infection. With Candida vaginitis,

pseudohyphae may be seen on a 10% potassium hydroxide preparation rather than on the wet mount. Epithelial cells are seen on the saline slide with physiologic leukorrhea. References: Emans SJ. Vulvovaginal complaints in the adolescent. In: Emans SJH, Laufer MR, Goldstein DP, eds. Pediatric and Adolescent Gynecology. 4th ed. Philadelphia, Pa: Lippincott-Raven Publishers; 1998:423-456 Gittes EB, Irwin CE Jr. Sexually transmitted diseases in adolescents. Pediatr Rev. 1993;14:180-189 Sobel JD. Vaginitis. N Engl J Med. 1997;337:1896-1903 Vandeven AM, Emans SJ. Vulvovaginitis in the child and adolescent. Pediatr Rev. 1993;14:141-147 Critique 231 Preferred Response: E

[View Question] Although the prediction of adult height is difficult, it can be a very important concern for the adolescent. Several methods are available to estimate adult height, although many require the knowledge of parental height and skeletal maturation of the adolescent. Formulas have been developed to predict adult heights with the knowledge that most individuals have an adult height that is within 5.08 cm (2 in) of the midparental height. For girls, the following formula can be used to calculate this information (father's height [cm] - 13) + mother's height (cm) 2 A more simplistic formula is to average the parents' heights and then subtract 6.5 cm for girls. In addition to these methods for estimating ultimate height, tables have been designed to predict height using current height, chronologic age, and skeletal age. Other tables use the adolescent's sexual maturity rating to predict adult height. Several programs for height prediction now are available for use on personal computers. Because parental information is not always available or may not be accurate, specific facts regarding growth and development in the adolescent female may be useful for estimating final adult height. There is little or no correlation between adult height and either age of onset of the growth spurt, age of peak height velocity (PHV), or height gain during puberty. There does seem to be a correlation, however, between adult height and the height at the onset of the growth spurt or height at PHV. Another important point to remember is that girls' heights rarely increase more than 5.08 cm (2 in) after menarche has occurred, and menarche always occurs after the PHV has been attained.

It is incorrect to state that adult height cannot be predicted or that one parent contributes more to final adult height than the other. In addition, neither the age at the onset of puberty nor the amount of sleep during puberty affects final height. References: Forbes GS. Nutrition and growth. In: McAnarney ER, Kreipe RE, Orr DP, Comerci GD, eds. Textbook of Adolescent Medicine. Philadelphia, Pa: WB Saunders Co; 1992:68-74 Kreipe RE. Normal somatic adolescent growth and development. In: McAnarney ER, Kreipe RE, Orr DP, Comerci GD, eds. Textbook of Adolescent Medicine. Philadelphia, Pa: WB Saunders Co; 1992:44-67 Neinstein LS, Kaufman FR. Normal physical growth and development. In: Neinstein LS, ed. Adolescent Health Care: A Practical Guide. 3rd ed. Baltimore, Md: Williams & Wilkins; 1996:9-11 Critique 256 Preferred Response: B

[View Question] Painful menstruation (dysmenorrhea) is one of the most common gynecologic complaints of young women. It has been estimated that 50% to 75% of women have some degree of menstrual discomfort, with as many as 10% experiencing severe, incapacitating pain for 1 to 3 days each month. Treatment is directed at symptomatic relief. Prior to initiating therapy, however, a distinction must be made between primary and secondary dysmenorrhea. Primary dysmenorrhea is menstrual pain that is not associated with any pelvic pathology. It is the most common form of dysmenorrhea among young women and is related to the increased production and release of prostaglandins by the endometrium, which causes uterine contractions. When released into the systemic circulation, endometrial prostaglandins cause headache, nausea, vomiting, and diarrhea. Typically, primary dysmenorrhea begins 1 to 3 years after menarche, once menstrual cycles have become ovulatory. Symptoms include crampy lower abdominal pain during the first few days of the menstrual cycle that may last for 24 to 48 hours. The pain can radiate to the lower back and thighs. Occasionally, the pain may begin a few days prior to menstruation and last 3 to 4 days into the cycle. Secondary dysmenorrhea is defined as painful menses resulting from underlying pelvic pathology. Possible etiologies include endometriosis, pelvic inflammatory disease, genital tract obstruction (eg, cervical stenosis or strictures), congenital malformations (eg, bicornate uterus with partial obstruction), uterine fibroids or polyps, ovarian neoplasms, and the presence of intrauterine devices. Clues to the diagnosis of secondary dysmenorrhea are a history of pelvic pain at times other than menstruation, intermenstrual bleeding, menorrhagia, and a history of sexual activity or pelvic infections. Painful menstruation from the onset of menarche suggests the presence of an obstruction in the genital tract.

First-line treatment for primary dysmenorrhea includes the appropriate use of drugs that inhibit prostaglandin synthesis, such as ibuprofen and naproxen. Another nonsteroidal anti-inflammatory drug (NSAID) that can be used is mefenamic acid, which also prevents prostaglandin action by binding to the prostaglandin receptor sites. NSAIDs should be prescribed at adequate doses, taken at the first sign of abdominal cramping or bleeding, and used for the duration of the symptoms. Because gastrointestinal irritation is the most common side effect, patients should be instructed to take the medication on a full stomach. Antiprostaglandin medications are 70% to 90% effective in the treatment of primary dysmenorrhea and usually are prescribed for two to three cycles to determine their efficacy before changing to another preparation. If the adolescent is sexually active and desires birth control or if NSAIDs are ineffective, a course of combination estrogen/progestin oral contraceptives (OCs) can be initiated. OCs prevent ovulation and cause endometrial hypoplasia, thereby reducing the amount of prostaglandin produced. Failure of adequate medical therapy warrants the search for pelvic pathology and further diagnostic tests such as laparoscopy. The administration of acetaminophen is not appropriate management for primary dysmenorrhea because it has no effect on prostaglandin production. Similarly, dietary modification and increased exercise are not of particular benefit. Reassurance that pain is a normal part of menstruation is inappropriate. References: Braverman PK, Sondheimer SJ. Menstrual disorders. Pediatr Rev. 1997;18:17-26 Laufer MR, Goldstein DP. Dysmenorrhea, pelvic pain, and premenstrual syndrome. In: Emans SJH, Laufer MR, Goldstein DP, eds. Pediatric and Adolescent Gynecology. 4th ed. Philadelphia, Pa: Lippincott-Raven Publishers; 1998:363-410 O'Connell BJ. The pediatrician and the sexually active adolescent. Treatment of common menstrual disorders. Pediatr Clin North Am. 1997;44:1391-1404 Polaneczky MM, Slap GB. Menstrual disorders in the adolescent: dysmenorrhea and dysfunctional uterine bleeding. Pediatr Rev. 1992;13:83-87 1999 Self-Assessment Exercise XXIV. Adolescent medicine and gynecology [Return to Category List] Questions [Print Directions] Question 14. Answer.

A 17-year-old girl comes to your office seeking a method of contraception. You discuss both barrier and hormonal methods with her. She inquires about complications associated with each method.

Of the following, the statement you are MOST likely to include in your discussion is that A. B. C. D. E. condoms with spermicide increase the risk of fetal anomalies intrauterine devices increase the risk of pelvic inflammatory disease long-acting injectable progestins are associated with a weight gain of at least 20 lb oral contraceptives increase the risk of uterine cancer subdermal progestin implants improve acne but may impair blood coagulation Answer.

Question 62.

A previously healthy 13-year-old girl has secondary amenorrhea. Menarche occurred at 12 years of age and periods occurred monthly until approximately 3 months ago. She has Tanner or sexual maturity rating (SMR) 4 breast development and SMR 3 pubic hair development. Findings on physical examination are unremarkable. Urine pregnancy test is negative. Of the following, the NEXT step in the management of this patient who has secondary amenorrhea is to A. B. C. D. E. measure serum androgen levels measure serum gonadotropin levels measure serum thyroxine concentration obtain pelvic ultrasonography schedule follow-up in 3 months

Question 107. Answer. An 18-year-old boy comes to your office because a known sexual contact was recently diagnosed with trichomoniasis and genital warts. Of the following, the finding MOST likely to be present on physical examination is A. B. C. D. E. a cluster of shallow, tender ulcers at the corona of the penis normal genitalia with no lesions or discharge papules with a dome-shaped top and central umbilication on the penis purulent discharge from the urethra tender inguinal adenopathy bilaterally

Question 155. Answer.

A 17-year-old girl complains of bumps on her labia. She is taking is oral contraceptives, but no other medications. Pelvic examination is unremarkable except for a few small warts at the introitus. You obtain endocervical cultures for Neisseria gonorrhoeae and Chlamydia trachomatis. Of the following, the MOST appropriate additional study is A. B. C. D. E. biopsy of lesions colposcopy microscopic urinalysis Papanicolaou smear pelvic ultrasonography

Question 205. Answer. A 15-year-old girl who recently has become sexually active complains of dysuria for the past 3 days. Physical examination reveals tender inguinal adenopathy bilaterally and multiple painful ulcers on the labia majora. After obtaining appropriate cultures and serologic tests to confirm the etiology of these findings, the most appropriate INITIAL management is administration of A. B. C. D. E. a single dose of intramuscular ceftriaxone a single dose of oral azithromycin oral acyclovir for 7 days oral doxycycline for 21 days topical acyclovir ointment to the lesions for 7 days

Question 246. Answer. Physical examination of a 17-year-old girl reveals swelling of both the parotid and submandibular glands, epigastric tenderness, and intermittent edema. Serum electrolyte concentrations include: sodium, 131 mEq/L; potassium, 2.3 mEq/L; chloride, 90 mEq/L; and bicarbonate, 32 mEq/L. Of the following, the best INITIAL step in the management of this adolescent is to A. B. C. D. ask her about her eating behaviors measure the serum amylase concentration obtain a consultation for upper endoscopy order a barium swallow

E.

prescribe oral potassium supplements

Answers Critique 14 Preferred Response: B

[View Question] The two primary methods of contraception used by adolescents to prevent pregnancy are hormonal and barrier methods. A combined regimen of both methods (eg, condoms plus oral contraceptives), which provides protection against sexually transmitted diseases (STDs) as well as pregnancy, is optimal. The advantages and disadvantages or complications associated with each method should be explained to the adolescent to improve adequate compliance. Barrier methods of contraception include condoms, diaphragms, vaginal spermicides, cervical caps, and the female condom. Vaginal contraceptive sponges are no longer available. Both male and female condoms act as mechanical barriers to prevent sperm from entering the vagina, serve as receptacles for seminal fluid, and help prevent the transfer of bacterial and viral organisms because there is no direct contact between the penis and the vagina. Condoms are available with lubrication as well as a spermicide. Advantages include easy accessibility, the ability to prevent STDs, and the relative ease of usage. There are no significant risks to condom use, except for occasional allergic reactions to the latex, lubricants, or spermicide. Condoms with spermicide have not been associated with an increased risk of fetal anomalies. The diaphragm and cervical caps are not recommended for adolescents because correct usage requires motivation and some comfort with inserting the devices. The intrauterine device (IUD), although a very effective method of contraception, is inappropriate for adolescents because its use is associated with an increased risk of pelvic inflammatory disease, a particular risk among teens, who generally are not monogamous. Hormonal contraceptives include combined oral contraceptives (which contain a combination of estrogen and progesterone), the progestin-only pill, injectable progestins, and implantable progestins. Combined oral contraceptives prevent pregnancy by modifying the patterns of ovulation, implantation, and menstruation. Although adverse effects and risks associated with oral contraceptive usage in young, healthy, nonsmoking females are minimal, adolescents should be monitored closely for rare serious adverse effects such as thromboembolism as well as for compliance. Noncontraceptive benefits of oral contraceptives include a decreased risk of ovarian and endometrial cancer, ovarian cysts, and menstrual disorders (dysmenorrhea, menorrhagia). The progestin-only pill can be offered to lactating adolescents who wish to use oral hormonal contraception because it will not decrease milk production. It also may be used in adolescents who experience estrogen-related side effects. However, the progestin-only pill prevents ovulation in only 15% to 40% of cycles and works primarily by altering cervical mucus, tubal mobility, and the endometrial lining. The major side effects are unpredictable menstrual patterns and breakthrough bleeding in approximately 10% to 30% of users.

Long-acting injectable progestins are a convenient and effective form of contraception for adolescents. The most common side effect is initial irregular menstrual bleeding followed by amenorrhea with prolonged usage. Infrequent side effects include minimal weight gain, hair loss, and acne. Subdermal progestin implants are highly effective. Side effects are similar to those of the injectable progestins, except that a minor surgical procedure is required to insert the plastic rods. Progestins do not affect carbohydrate metabolism, blood coagulation, or liver function. Minor side effects include headache, acne, weight gain, and infection at the site of implantation. Informed consent is required prior to insertion. References: Rosenfeld WD, Swedler JB. Role of hormonal contraceptives in prevention of pregnancy and sexually transmitted diseases. Adolescent Medicine: State of the Art Reviews. 1992;3:207222 Sikand A, Fisher M. The role of barrier contraceptives in prevention of pregnancy and disease in adolescents. Adolescent Medicine: State of the Art Reviews. 1992;3:223-240 Critique 62 Preferred Response: E

[View Question] The average age of menarche in the United States is approximately 12.8 years, with AfricanAmerican females reaching menarche a few months before Caucasians. The timing of pubertal development is relatively predictable: 5% of adolescents have menarche when they reach sexual maturity rating (SMR) 2, 25% in SMR 3, 60% in SMR 4, and 10% in SMR 5. Thus, nearly 90% of females will have begun to menstruate before reaching SMR 5 breast and pubic hair development. Further, menarche usually occurs 2 to 2.5 years after the beginning of breast development or thelarche and 1 year after the growth spurt (peak height velocity). Although the age and timing of menarche is relatively predictable in females, menstrual patterns can be quite variable in the first 2 years following menarche because most early menstrual cycles are anovulatory. Therefore, for a few years following menarches, it is common for an adolescent to experience irregular menstrual bleeding patterns, such as several months of amenorrhea or intermittent prolonged menses. The positive feedback mechanism required for ovulation does not develop until middle to late puberty when rapidly rising levels of estrogen trigger the leutinizing hormone surge necessary for a normal cycle. Thus, the development of regular, ovulatory cycles may not occur until several years following menarche, and about 10% to 20% of cycles can remain anovulatory for up to 5 years. Under most circumstances, infrequent or irregular menstrual cycles in a healthy adolescent within the first 2 years following menarche, such as described in the vignette, do not warrant laboratory investigation. However, because pregnancy is the most common cause of secondary amenorrhea in an adolescent, it is appropriate to order a urine pregnancy test at the initial visit. Once pregnancy has been excluded, the adolescent should

be reassured that the irregular pattern of her menses is common at this age and she should be re-evaluated in 3 to 6 months. Measurement of serum androgen levels is appropriate for the obese adolescent who has signs of virilization, such as hirsutism and acne. Measurement of serum gonadotropin levels is not warranted in the girl in the vignette because her appropriate pubertal development is evidence of an intact hypothalamic-pituitary-ovarian axis. In the absence of signs and symptoms of hypo- or hyperthyroidism, measurement of a serum thyroxine concentration would not be indicated. Pelvic ultrasonography should be performed if the complaint were primary amenorrhea to exclude congenital anomalies such as absence of the uterus or imperforate hymen. However, pelvic ultrasonography is not warranted in the girl described in the vignette, who has secondary amenorrhea. References: Braverman PK, Sondheimer SJ. Menstrual disorders. Pediatrics in Review. 1997;18:17-25 Mansfield MJ. Normal female reproductive development and amenorrhea. In: McAnarney ER, Kreipe RE, Orr DP, Comerci GD, eds. Textbook of Adolescent Medicine. Philadelphia, Pa: WB Saunders Co; 1992:659-666 Polaneczky MM, Slap GB. Menstrual disorders in the adolescent: amenorrhea. Pediatrics in Review. 1992;13:43-49 Critique 107 Preferred Response: B

[View Question] Genital warts or condyloma acuminatum are probably the most common viral sexually transmitted disease (STD) in the United States. These warts, which also can be found in the anal area, are caused by human papillomavirus (HPV). Infections can be symptomatic, subclinical, or asymptomatic in both males and females, with 17% of untreated anogenital warts disappearing spontaneously by 16 weeks and 70% by 2 years. In men, HPV infection produces common and plantar warts, condyloma acuminatum, and subclinical condylomata, also known as flat condylomata. Classic condylomata are soft, flesh-colored lesions that have an irregular surface and are 2- to 5-mm in diameter. They commonly occur on the coronal sulcus of the penis, inside the prepuce, and at the frenulum. Flat condylomata are areas of either normal-appearing or slightly elevated skin that turn white with the application of 5% acetic acid. On biopsy, these subclinical lesions show a histologic pattern consistent with HPV. Genital warts are strongly associated with penile and anal carcinoma. In women, in addition to the previously described lesions, infection with HPV produces squamous intraepithelial lesions (formerly known as cervical intraepithelial neoplasia), a precursor of cervical carcinoma. Because of this strong association between HPV and cervical carcinoma, it is recommended that patients who have HPV infections be monitored closely by means of frequent Papanicolaou (Pap) smears.

Other lesions occurring in adolescents that look similar to condyloma acuminatum and can be confused easily with genital warts include molluscum contagiosum and "pink pearly papules". Molluscum contagiosum consists of 3- to 5-mm papules that have a domeshaped top and central umbilication. "Pink pearly papules" are 1- to 3-mm lesions that occur in rows at the corona of the penis in approximately 15% of normal pubertal males. Trichomoniasis is caused by the motile, flagellated protozoon, Trichomonas vaginalis, and is acquired through sexual contact. In women, infection produces a yellowgreen, frothy, malodorous vaginal discharge that often is associated with intense pruritus. The diagnosis can be made by microscopic examination of the vaginal discharge, which demonstrates the motile trichomonads. In males, T vaginalis is responsible for some cases of nongonococcal, nonchlamydial urethritis manifested by a scant clear discharge that may be overlooked by the patient. Asymptomatic infections also have been documented, making males an important reservoir for the transmission of this infection. Accordingly, an adolescent male who has been exposed to trichomoniasis and genital warts, such as the boy in the vignette, is most likely to have normal results on physical examination. A purulent urethral discharge in a male classically is associated with gonococcal infection, but it also may be caused by Chlamydia trachomatis. A cluster of shallow, tender penile ulcers and inguinal adenopathy are most likely due to herpes simplex infection. References: Dyment PG. Human papillomavirus infection. Adolescent Medicine: State of the Art Reviews. 1996;7:119-129 Krowchuk DP. Nongonococcal urethritis: diagnosis, management, and complications. Adolescent Medicine: State of the Art Reviews. 1996;7:63-82 Rosenfeld WD. Sexually transmitted diseases in adolescents: update 1991. Pediatr Ann. 1991;20:303-312 Critique 155 Preferred Response: D

[View Question] All sexually active adolescent females should undergo routine Papanicolaou (Pap) screening because it can detect cervical dysplasia, which is associated with human papillomavirus (HPV), a sexually transmitted pathogen. Cervical cancer begins as cervical dysplasia, develops into carcinoma in situ, and finally becomes invasive cancer. Early detection is imperative because appropriate evaluation and treatment of cervical dysplasia can eliminate progression to cervical cancer. Current guidelines recommend Pap screening for sexually active women at any age. In addition, Pap screening should be started at age 18 even if there is no history of sexual activity. In general, a Pap smear should be repeated every 3 years in adults and probably annually in sexually active adolescents.

Indications for a more frequent Pap smear in young females include first coitus before age 18; multiple sexual partners; partners who have multiple sexual contacts; a history of sexually transmitted diseases (STDs), including HPV infection; and a prior abnormal Pap smear. In such adolescents, routine Pap screening may be indicated more frequently at the discretion of the physician. Smoking, use of oral contraceptives, and nonuse of barrier contraceptive methods are factors that place the adolescent at increased risk for acquiring a STD. Exposure to diethylstilbestrol in utero is another indication for more frequent Pap smear screening, although most women who were exposed to this medication are no longer adolescents. Biopsy of genital warts is indicated only if the diagnosis is uncertain. If the Pap smear is abnormal, colposcopy is indicated. Pelvic ultrasonography is of no value in the assessment of warts. Although urinalysis (eg, detection of leukocyte esterase activity) may be useful in the diagnosis of urethritis in males, it is of limited value in females. References: Beach RK. Why teenage girls need yearly Pap smears. Contemporary Adolescent Gynecology. 1997;2:4-14 Neinstein LS, Schubiner HH. Health screening and evaluation. In: Neinstein LS, ed. Adolescent Health Care: A Practical Guide. Baltimore, Md: Williams & Wilkins; 1996:61-109 Critique 205 Preferred Response: C

[View Question] The most common cause of painful genital ulcers in a sexually active adolescent is herpes simplex virus (HSV). Infection with HSV-2 usually results from direct contact with infected genital secretions or lesions as a result of sexual activity, whereas HSV-1 infections usually occur from direct contact with infected oral secretions or lesions. Prior teaching that HSV-2 usually is not the cause of lesions "above the waist" is no longer valid; both strains can be recovered from the pharynx and the genital tract as a result of oral-genital sexual activity. Although primary genital HSV infection may be asymptomatic, most primary infections are associated with more severe signs and symptoms than recurrent infections. Primary infections often are accompanied by systemic symptoms (eg, fever, headache, malaise, abdominal pain), and lesions can be papular, vesicular, or ulcerative and involve an extensive area. Localized symptoms may include pain, itching, dysuria, and urethral or vaginal discharge. Tender inguinal adenopathy and extragenital lesions also may occur. Viral shedding is more prolonged in patients who have a primary genital HSV infection compared with those who have recurrent infections. Most women (>80%) who have a symptomatic primary genital HSV infection have cervical and urethral involvement and, if untreated, can shed the virus for up to 3 months even though lesions may be visible for only a few weeks. Urinary retention due to pain or autonomic nerve involvement also may occur. This complication can be relieved by frequent urethral catheterization and adequate pain medication.

In adults and adolescents who have symptomatic primary genital HSV infection, treatment with acyclovir diminishes the duration of symptoms as well as viral shedding. Oral acyclovir treatment initiated within 6 days of the onset of symptoms decreases viral shedding and new lesion formation by approximately 3 to 5 days and decreases symptoms. It does not, however, affect the number or frequency of subsequent recurrences. The dose of oral acyclovir recommended for genital HSV infection is 200 mg five times a day or 400 mg three times a day for a total of 7 to 10 days. Topical acyclovir ointment only minimally reduces viral shedding and symptoms in patients who have genital HSV infection and is not recommended. Antimicrobials such as ceftriaxone, azithromycin, and doxycycline are neither effective nor appropriate therapy for HSV. The results of serologic tests and microscopic evaluation of cervical secretions as well as cultures should determine whether any antimicrobials are needed to treat a concomitant sexually transmitted disease (eg, chlamydia, gonorrhea, syphilis). References: Annunziato PW, Gershon A. Herpes simplex virus infections. Pediatrics in Review. 1996;17:415-423 Bryson YJ. Genital herpes in adolescents and young adults. Adolescent Medicine: State of the Art Reviews. 1990;1:471-496 Committee on Infectious Diseases. Herpes simplex. In: Report of the Committee on Infectious Diseases. 24th ed. Elk Grove Village, Ill: American Academy of Pediatrics; 1997:266-276 Critique 246 Preferred Response: A

[View Question] Bulimia nervosa is an eating disorder that is characterized by specific behavioral, affective, and cognitive components. Simply defined, the key clinical feature of bulimia is binge eating. To make the diagnosis, however, formal criteria have been established and are listed in the Diagnostic and Statistical Manual of Mental Disorders, 4th edition (DSM-IV). Behavioral criteria include recurrent episodes of binge eating, which is defined as consuming an amount of food larger than most people would eat in a discrete period, in addition to some inappropriate compensatory activity to prevent weight gain, such as self-induced vomiting, laxative abuse, misuse of diuretics or enemas, fasting, or vigorous exercise. The affective component of this disorder is the lack of control during the binging episodes followed by depression or self-deprecating thoughts; cognitive criteria are based on the adolescent's overconcern with body shape and weight. Both the binge eating and inappropriate compensatory behaviors should occur at least twice a week for 3 months to meet the diagnosis of bulimia nervosa. Of the options listed, the best initial step in managing an adolescent who is suspected of having an eating disorder would be to ask her about her eating behaviors to see if these diagnostic criteria are met. Classic clinical manifestations of this illness are: salivary gland enlargement secondary to stimulation by binge eating and vomiting, dental enamel erosion by gastric

acids as a result of chronic vomiting, bruises or calluses over the dorsum of the second to fifth metacarpal-phalangeal joints due to self-stimulation of the gag reflex, and wide fluctuations in body weight with intermittent lower extremity edema. Electrolyte abnormalities can be caused by chronic vomiting or medication misuse, although these levels also may be normal. A hypokalemic hypochloremic metabolic alkalosis is a typical finding in patients who have a history of chronic vomiting, whereas laxative abuse may be associated with acidosis. The management of an adolescent who has an eating disorder should be multidisciplinary, involving the primary pediatrician, a psychologist to provide family therapy or a psychiatrist to provide individual psychotherapy and perhaps medication, and a nutritionist. Especially for adolescents who require hospitalization, the expertise of an adolescent medicine physician may be warranted. Even if the adolescent does not meet all of the criteria for a classic eating disorder, intervention is recommended for those who have a distorted body image, weigh 90% or less than average for height, or use harmful weight control measures. Mild or early cases often can be treated successfully on an outpatient basis. Indications for hospitalization of the adolescent who has bulimia include: failure of outpatient treatment, especially if the medical condition deteriorates; dehydration; electrolyte disturbances such as hypokalemia; electrocardiographic abnormalities (eg, prolonged QTc interval, severe bradycardia, dysrhythmias, or features of hypokalemia); acute food refusal; uncontrollable binge eating and purging; and acute medical complications (eg, cardiac failure, Mallory-Weiss esophageal tear, gastric rupture). Acute psychiatric emergencies, such as suicidal ideation or acute psychosis, or evidence of a comorbid diagnosis that interferes with the treatment of the eating disorder (eg, severe depression, substance abuse, obsessive compulsive disorder) also are indications for inpatient treatment of adolescents who have eating disorders. Endoscopy or barium swallow might be pursued if there was evidence of specific gastrointestinal pathology, and serum amylase levels might be obtained if there was evidence of pancreatic pathology. However, these tests should not be ordered before taking a thorough history from the patient. Hypokalemia is one of the indications for hospitalization of an adolescent who has an eating disorder. Outpatient prescription of oral potassium supplement would not be an appropriate initial intervention. References: Fisher M. Medical complications of anorexia and bulimia nervosa. Adolescent Medicine: State of the Art Reviews. 1992;3:487-502 Harper G. Eating disorders in adolescence. Pediatrics in Review. 1994;15:72-77 Kreipe RE. Eating disorders among children and adolescents. Pediatrics in Review. 1995;15:370-379 Stashwick C. When you suspect an eating disorder. Contemporary Pediatrics. 1996;13:124153

1998 Self-Assessment Exercise XXIV. Adolescent medicine and gynecology [Return to Category List] Questions [Print Directions] Question 18. Answer.

A 16-year-old girl was sexually assaulted 3 months ago. Since then, she has had recurring nightmares, is fearful of walking alone to school, and has been disinterested in her school activities. She also complains of lower abdominal pain, decreased appetite, and vaginal discharge. Of the following, the MOST likely cause of her symptoms is A. B. C. D. E. major depression occult genital trauma pelvic inflammatory disease rape-trauma syndrome somatoform disorder Answer.

Question 42.

A 16-year-old girl who has sickle cell disease wishes to use oral contraceptives. She had a stroke 3 years ago associated with persistent left-sided weakness and a seizure disorder. Menarche occurred at age 14 and menses are irregular. She has been sexually active for 2 years and has had three partners. Her current partner occasionally uses condoms. The most important CONTRAINDICATION to use of oral contraceptives in this patient is her history of A. B. C. D. E. epilepsy inconsistent use of condoms irregular menstrual bleeding multiple sexual partners stroke Answer.

Question 63.

A 16-year-old girl who dropped out of high school last year has not menstruated for 3 months. She has been sexually active with one partner for 1 year. She has not been using any contraception. Results of a urine pregnancy test are positive.

Among the following, the MOST likely reason that this adolescent has failed to use contraception is A. B. C. D. E. concern about confidentiality concern about weight gain cost of contraception desire to become pregnant partner opposition to contraceptive use Answer.

Question 84.

The mother of a 13-year-old boy is concerned because her son has started to associate with a group of older boys whom she suspects belong to a gang. Of the following, the statement you are MOST likely to include in your discussion with the mother about adolescent friendships is that A. B. adolescents usually seek parental approval of their friends friends have a powerful influence on an adolescent's risk-taking behaviors

C. adolescents dress differently from their friends as a way of establishing their independence D. older adolescents are more likely than younger adolescents to spend time with friends of the same gender E. older adolescents prefer friendships with younger adolescents

Question 104. Answer. The three LEADING causes of death among adolescents in the United States are A. B. C. D. E. accidents, cardiovascular disease, malignancy accidents, homicide, suicide cardiovascular disease, homicide, suicide homicide, malignancy, suicide homicide, infections, malignancy

Question 127. Answer. The mother of an 11-year-old girl asks you to describe the stages of adolescent growth and sexual development so that she can prepare her daughter. Of the following, the information that you are MOST likely to include in the discussion is that

A. B. C. D. E.

menarche occurs 2 years after thelarche peak height velocity occurs 1 year after menarche pubarche is the first sign of secondary sexual development the average age at menarche is 14 years the average age at thelarche is 12 years

Question 147. Answer. The mother of a 13-year-old boy wants her son to be screened for elevated cholesterol levels. Family history reveals that the father, who has hypercholesterolemia, recently had a myocardial infarction at age 48. There is no other history of heart disease in the family. Of the following, your BEST advice about the boy's cholesterol level is that it A. B. C. D. E. is affected by his diet and weight is not affected by pubertal development is predictive of his adult level should be approximately 210 mg/dL should not be measured until he is an adult

Question 169. Answer. A previously healthy 14-year-old boy is concerned because his height, which has increased only 7 cm in the past year, is 167.6 cm (5 ft 6 in). Physical examination reveals absent facial hair, bilateral gynecomastia, a sexual maturity rating of 4 for pubic hair and 3 for penile growth, and testes that are 2.0 cm in length. Of the following, the MOST important reason for further evaluation of this boy's pubertal development is A. B. C. D. E. abnormal testicular development absence of facial hair bilateral gynecomastia decreased height velocity for age disparity between pubic hair and penile development

Question 189. Answer. A 15-year-old girl has had amenorrhea for 3 months. Menarche occurred at age 12. She is an A student and is on the school track team. Physical examination reveals height at the 50th

percentile for age, weight at the 5th percentile, and a sexual maturity rating of 5. A urine pregnancy test is negative. You suspect an eating disorder. The diagnosis of anorexia nervosa is MOST likely if she A. B. C. D. E. exercises 2 hours each day feels sad and worthless has been dieting for 6 months takes diuretics to control her weight thinks she is fat

Question 211. Answer. A 15-year-old sexually active girl has had irregular, heavy menstrual bleeding since menarche at age 12. She has been well except for prolonged bleeding after a tooth extraction. Findings include a hand-shaped bruise on the left shoulder; moderate bleeding from the cervix without clots; hemoglobin, 8 g/dL; white blood cell count, 10,000/mm; platelet count, 300,000/mm; and a negative urine pregnancy test. Of the following, the MOST likely diagnosis is A. B. C. D. E. blood dyscrasia pelvic inflammatory disease physiologic menorrhagia tubal pregnancy vaginal trauma

Question 232. Answer. You are discussing how to determine sexual maturity rating (SMR) with a medical student who is observing in your office. Among the following, the statement about the sequence of development of secondary characteristics you are MOST likely to include in your discussion is that A. B. C. D. E. breast and pubic hair development in girls usually occur within the same SMR stage penile enlargement usually occurs before testicular enlargement pubic hair development usually is a first sign of puberty in boys SMR 3 is characterized by extension of pubic hair onto the thighs SMR 4 is characterized by gynecomastia in boys and darker areolae in girls

Question 254. Answer. A 16-year-old previously healthy girl complains of severe lower abdominal pain that began 2 days ago. Her last menstrual period was 3 weeks ago. She is sexually active with one partner. On physical examination, she is afebrile and has lower abdominal tenderness, but no palpable abdominal mass. Results of a urinalysis and white blood cell count are normal. The INITIAL study that should be performed in this patient is A. B. C. D. E. abdominal radiography abdominal ultrasonography measurement of serum amylase activity pelvic examination urine culture

Answers Critique 18 Preferred Response: D

[View Question] An adolescent who is fearful, apathetic, and has sleep disturbances following a sexual assault most likely is suffering from rape-trauma syndrome, one type of posttraumatic stress disorder (PTSD). Approximately one third of sexual abuse victims have symptoms consistent with PTSD following an assault. A variety of other extremely traumatic events, such as violent physical attacks, kidnapping, life-threatening accidents, and natural or man-made disasters, also can cause symptoms compatible with PTSD. These symptoms include intense fear, continued re-experiencing of the traumatic event, ongoing avoidance of stimuli associated with the trauma, and persistent symptoms of increased arousal. Symptoms may develop immediately after the traumatic event or be delayed in onset. The fears experienced with PTSD may be related to the circumstances surrounding the traumatic event. For example, if a girl was raped while walking alone, she now may be fearful of being outdoors alone. Conversely, if the assault occurred in the home, she may be fearful of being home alone. The traumatic event may be re-experienced in a variety of ways, including intrusive thoughts and recurring nightmares. Intense psychologic distress or physiologic reactivity often occurs when the person is exposed to triggering events that resemble or symbolize an aspect of the traumatic event (eg, anniversaries of the event or visiting locations that are reminiscent of the site of the traumatic event). Stimuli associated with the trauma are avoided persistently by the victim. Thus, the person commonly avoids thoughts, feelings, or conversations about the traumatic event as well as activities, situations, and people who arouse recollections of the trauma. Diminished interest in activities that previously were pleasurable is common. Symptoms of

anxiety also are noted, including sleep problems, hypervigilance, exaggerated startle response, and irritability. Symptoms of vaginal discharge and abdominal pain following a sexual assault may be due to sexually transmitted diseases, including pelvic inflammatory disease, or to occult trauma. It is important to determine if the victim was examined and tested for sexually acquired infections following the assault. Such victims should be evaluated for sexually transmitted diseases, including Neisseria gonorrhoeae, Chlamydia trachomatis, human immunodeficiency virus, and syphilis. Most often, persistent symptoms of lower abdominal pain and discharge following an assault are due to psychologic rather than organic causes. However, performing repeat cultures and serologic tests may be helpful adjuncts to counseling if there are persistent concerns about sexually transmitted diseases. Similarly, repeating a pelvic examination can help reassure the patient who is concerned about occult trauma; this would be unlikely unless there was significant physical trauma noted at the time of the initial evaluation. Individuals who have PTSD are at increased risk for other psychologic problems, including panic disorder, social phobia, specific phobia, major depressive disorder, somatization disorder, and substance-related disorders. Indeed, the symptoms of PTSD overlap with many other psychologic conditions. Because rape victims often do not report the assault due to fear and embarrassment, it is important for physicians to be aware of the symptoms of PTSD and consider this diagnosis whenever an adolescent has symptoms suggestive of it, even if there is no history of a traumatic event. Community-based studies have shown that the lifetime prevalence of PTSD from any cause ranges from 1% to 14%. With the increasing prevalence of violence in our society, physicians can expect to encounter growing numbers of children and adolescents who have PTSD. Recognition of this disorder by a physician can be the first step in a patient's psychologic recovery from the traumatic experience. Needless laboratory tests also can be avoided when somatic symptoms are recognized as part of PTSD rather than an organic disease, and the patient can be provided with appropriate reassurance and counseling. References: American Academy of Pediatrics Committee on Adolescence. Sexual assault and the adolescent. Pediatrics. 1994;94:761-765 Deblinger E, Lippmann J, Steer R. Sexually abused children suffering posttraumatic stress syndromes: initial treatment outcome findings. Child Maltreatment. 1996;1:310-321 Dunn SF, Gilchrist VJ. Sexual assault. Prim Care. 1993;20:359-373 Frances A, Pincus HA, First MB. Diagnostic and Statistical Manual of Mental Disorders-IV. Washington, DC: American Psychiatric Association; 1994:424-429

Pynoos RS, Nader K. Post-traumatic stress disorder. In: McAnarney ER, Kreipe RE, Orr DP, Comerci GD, eds. Textbook of Adolescent Medicine. Philadelphia, Pa: WB Saunders Co; 1992:1003-1009 Rimsza ME, Berg RA, Locke C. Sexual abuse: somatic and emotional reactions. Child Abuse Negl. 1988;12:201-208 Task Force on Adolescent Assault Victim Needs. Adolescent assault victim needs: a review of issues and a model protocol. Pediatrics. 1996;98:991-1001 Critique 42 Preferred Response: E

[View Question] An adolescent is almost 10 times more likely to die during childbirth than from complications of oral contraceptive use. Indeed, the risk of death from oral contraceptive use is 1.3/100,000 users compared with the risk of death during childbirth (11.1/100,000 live births) in this age group. There are only a few absolute contraindications to oral contraceptive use in adolescents, including thromboembolic disease, cerebrovascular accident, uterine cancer, coronary artery disease, and pregnancy. Undiagnosed genital bleeding also is a contraindication if reproductive tract malignancy is suspected. For teens, the most common contraindication is pregnancy; many adolescent girls only seek contraception when they fear they are pregnant. Indeed, failure to anticipate the need for contraception is one of the major obstacles to the effective use of contraceptives in adolescence. Sickle cell disease is not an absolute contraindication to the use of oral contraceptives. However, a patient who has sickle cell disease and a history of stroke is at increased risk for another cerebrovascular accident, and oral contraceptives will increase that risk. Thus, the history of stroke in the patient in the vignette is the most important contraindication to oral contraceptives. Strokes may be either hemorrhagic or thromboembolic. Oral contraceptive users also are two to four times more likely to develop thromboembolic disease, such as deep vein thrombosis and pulmonary embolism, than are nonusers. Epilepsy is not a contraindication to the use of oral contraceptives. However, many of the anticonvulsants commonly used to treat epilepsy, including phenytoin, carbamazepine, and barbiturates, increase the catabolism of oral contraceptives, thus decreasing their efficacy. Therefore, most physicians recommend another method of contraception for the adolescent who receives one of these anticonvulsants. Multiple sexual partners and inconsistent use of condoms increase a teenager's risk of acquiring a sexually transmitted disease. Oral contraceptives appear to decrease the risk of gonococcal salpingitis and cervicitis. Although the mechanism is unclear, this protective effect may be due to changes in the cervical mucus. It is important to counsel teens that the use of oral contraceptives will not prevent them from acquiring sexually transmitted diseases; they must use condoms consistently as well as oral contraceptives for this purpose.

Irregular menstrual bleeding in the adolescent usually decreases with the use of an oral contraceptive. Indeed, regular use of oral contraceptives decreases the incidence of many common menstrual disorders. For example, the incidence of dysmenorrhea is reduced by 63%, menorrhagia by 48%, irregular menstruation by 35%, and premenstrual tension by 29% in oral contraceptive users compared with nonusers. In most adolescents, irregular menstrual bleeding is due to erratic levels of estrogen and progesterone release secondary to anovulatory cycles. When oral contraceptives are taken regularly, the levels of sex hormones are normalized and irregular bleeding is decreased. In older women, especially those older than age 35, irregular menstrual bleeding may be due to reproductive tract malignancy. Thus, it is especially important for women in this age group to have a thorough evaluation before prescribing oral contraceptives for this indication. Approximately one in every eight women between the ages of 15 and 19 years becomes pregnant each year, a proportion that has changed little since 1970. The vast majority of adolescents who become pregnant did not intend to do so, but there are many obstacles to the effective use of contraception in this population. In addition to delay in seeking contraception after initiating sexual activity, adolescents often find it difficult to use a contraceptive consistently. Concerns about confidentiality and myths about the risks of contraception also limit effective use. In some cases, teens do not have the financial resources to obtain contraception. In others, there is pressure from sex partners to become pregnant. Finally, a small proportion of teens want to become pregnant because they have no other career goals. References: Brookman RR. Contraceptive technology and practice. In: Friedman SB, Fisher M, Schonberg SK, eds. Comprehensive Adolescent Health Care. St Louis, Mo: Quality Medical Publishing, Inc; 1992:1016-1025 Chaikind J, Shafer M-A. Contraception. In: Rudolph AM, Hoffman JIE, Rudolph CD, eds. Rudolph's Pediatrics. 20th ed. Stamford, Conn: Appleton & Lange; 1996:64-67 Greydanus DE, Patel DR. Contraception. In: McAnarney ER, Kreipe RE, Orr DP, Comerci GD, eds. Textbook of Adolescent Medicine. Philadelphia, Pa: WB Saunders Co; 1992:676-688 Hatcher RA, Trussell J, Stewart F, et al. Contraceptive Technology. 16th ed. New York, NY: Irvington Publishers, Inc; 1994 Litt IF. Contraception. In: Nelson WE, Behrman RE, Kliegman RM, Arvin AM, eds. Nelson Textbook of Pediatrics. 15th ed. Philadelphia, Pa: WB Saunders Co; 1996:551-552 Rosenfeld WD, Swedler JB. Role of hormonal contraceptives in prevention of pregnancy and sexually transmitted diseases. Adolescent Medicine: State of the Art Reviews. 1992;3:207222 Critique 63 Preferred Response: D

[View Question]

Approximately 65% of adolescent girls in the United States become sexually active before completing high school, and only 65% of these teens reported use of a contraceptive at first intercourse. Unfortunately, 50% of all initial pregnancies in adolescents occur in the first 6 months following initiation of intercourse; 20% occur in the first month. The average interval between initiating sexual relations and the first family planning visit for 15- to 24-year-old women is 23 months. Among teenagers who do use contraception, many do not use the method correctly or consistently. Indeed, fewer than 50% of sexually active teens who use condoms report using one with every episode of intercourse. Although there are many reasons for American teens not using contraception, one of the most common is a desire to become pregnant. Approximately one third of nonpregnant American adolescents have positive or ambivalent feelings about immediate childbearing. Indeed, in a recent study of 200 pregnant adolescents (average age, 16 years) in Denver, the two most frequently cited reasons for not using contraception were "I didn't mind getting pregnant" (20% of respondents) and "I wanted to get pregnant" (17% of respondents). Adolescents who had positive or ambivalent attitudes toward pregnancy did not differ from others with regard to age, insurance status, gravidity, or partner's age. Thus, the authors concluded that the absence of negative attitudes toward pregnancy rather than negative attitudes about contraception were a major factor in the adolescents' failure to use contraception. Other reasons for failure to use contraception include partner or peer pressure, concern about side effects, lack of confidentiality, and cost. Teens have many misconceptions about the side effects of contraceptives. In a recent study, weight gain was the most common side effect of oral contraception that was of concern to teens. In fact, significant weight gain is uncommon with oral contraceptives. It is, however, one of the most common side effects of the injectable contraceptive medroxyprogesterone. Because of its anabolic effects, a weight gain of approximately 1 kg (2.2 lb) annually can be expected among its users. In focus group sessions, adolescents have revealed a strong desire for privacy concerning their sexual activity and strong feelings of embarrassment when talking about sexual matters to physicians, parents, and peers. This concern about confidentiality may be an important reason for delay in seeking contraception, and their worries are not groundless. Approximately 22% of physicians refuse to provide contraceptive services to teens without parental consent. Adolescent pregnancy rates are considerably higher in the United States than in any other developed country. These increased pregnancy rates are not caused by a higher prevalence of sexual activity; the proportion of teens who have had sexual intercourse is similar in all countries except Canada, where the proportions are lower, and Sweden, which has a much higher proportion of sexually active teens than the United States. Some of the major differences in contraceptive services between Sweden and the United States are the cost and availability. Because inexpensive contraceptive services are limited in the United States, many teens use cheaper, less effective over-the-counter methods of birth control. The cost of oral contraceptives is $18 to $20 per cycle and

medroxyprogesterone costs $35 per injection. Thus, the annual cost for oral contraceptives is approximately $240 and for medroxyprogesterone is $140. These costs do not include the expense of physician services or screening tests. It is easy to understand that most teens cannot pay the full cost of contraceptives without parental support or other assistance. References: Costa FM, Jessor R, Fortenberry JD, Donovan JE. Psychosocial conventionality, health orientation, and contraceptive use in adolescence. J Adolesc Health. 1996;18:404-416 DuRant RH. Methodologic commentary: why pregnant adolescents say they did not use contraceptives prior to conception. J Adolesc Health. 1996;19:54-55 Hatcher RA, Trussell J, Stewart F, et al. Contraceptive Technology. 16th ed. New York, NY: Irvington Publishers, Inc; 1994 Rainey DY, Stevens-Simon C, Kaplan DW. Self-perception of infertility among adolescents. Am J Dis Child. 1993;147:1053-1056 Stevens-Simon C, Kelly L, Singer D, Cox A. Why pregnant adolescents say they did not use contraceptives prior to conception. J Adolesc Health. 1996;19:48-53 Critique 84 Preferred Response: B

[View Question] The need to conform to peer standards in dress, grooming, and behavior increases dramatically in early adolescence. Conformity pressures are strongest during the junior high years and level off as adolescents reach the latter part of high school. The peer group has a profound influence on the adolescent's behaviors and is one of many factors that can cause an adolescent to engage in risk-taking behaviors, such as joining a violent gang or participating in substance abuse or early sexual experimentation. Adolescents rarely seek parental approval of their choice of friends. Indeed, the peer group is based largely on an adolescent's choice of friends and their acceptance by the peers with whom they wish to associate. Adolescents are very aware of specific subgroups among their peers and frequently invent names to describe them (eg, nerds, jocks, druggies). These subgroups often choose similar styles of dress to identify themselves as part of a particular peer group. Thus, although teenagers are seeking autonomy from their parents, they temper this independence by accepting the standards of dress and behavior of their peer group during this transition to adulthood. By late adolescence, the peer group has less importance in an adolescent's life, and relationships with individual members of their age group predominate. The younger adolescent is more likely to spend time with friends of the same gender, but in later adolescence close friendships often are established with members of the opposite gender. In all stages of adolescence, friendships tend to be with those of the same age. However, adolescents who mature early may seek out older teens who have a similar appearance, skills, and interests.

Teenagers will seek membership in groups that they consider to be of "high status," but such membership often is restricted to those who have similar interests, social characteristics, and physical appearance. Thus, for some adolescents the peer group they seek will not be open to them; they involuntarily will become part of a lower status group rather than suffer isolation. Although many adults assume peer influences among adolescents are unilaterally negative, they can produce a positive, socializing effect. Peer groups offer a context within which the young person can experiment with alternative concepts of oneself, learn social skills, and clarify value systems. Just as peers can exert pressure to engage in unhealthy behaviors, they also can exert pressures that promote positive health behaviors. Adolescents who lack friends are deprived of close ties with peers at a time when peer support is an essential resource for personal health and development. Indeed, adolescents who are friendless or rejected by peers are especially vulnerable to emotional and affective disorders, including suicide. Parents frequently ask pediatricians for advice about their adolescent's peer relations and activities. It is important for physicians to help parents understand the important role of peer groups in the social and emotional development of their teenager. Although parents cannot choose friends for their adolescent, they can provide opportunities to participate in groups that will foster social and emotional health. Also, by allowing the adolescent greater independence with maturity, there will be less need to rebel against family values in an attempt to establish autonomy. In one sense, all emotionally healthy teenagers belong to a "gang" of peers; problems occur when this group engages in socially unacceptable behavior. If a 13-year-old is associating with older teens who are engaging in criminal behavior, such as the mother in the vignette suspects, early intervention is essential. The boy may have become part of the gang as a result of rejection by other peers, depression, poor self-esteem, fear, or other familial factors. Redirecting his social activities before he becomes entrenched in the activities of the gang is extremely important. References: Cavanaugh RM. Anticipatory guidance for the adolescent: has it come of age?02383 Pediatrics in Review. 1994;15:485-489 Cox RP. An exploration of the demographic and social correlates of criminal behavior among adolescent males. J Adolesc Health. 1996;19:17-24 Ladd GW. Peer relationships. In: Friedman SB, Fisher M, Schonberg SK, eds. Comprehensive Adolescent Health Care. St Louis, Mo: Quality Medical Publishing, Inc; 1992:674-677 Weiner IB. Normality during adolescence. In: McAnarney ER, Kreipe RE, Orr DP, Comerci GD, eds. Textbook of Adolescent Medicine. Philadelphia, Pa: WB Saunders Co; 1992:86-98 Critique 104 Preferred Response: B

[View Question] The three leading causes of death among adolescents in the United States today are accidents, homicide, and suicide. Experts in injury prevention prefer that the word "accident" be replaced by the term "unintentional injury" to emphasize the preventability of these deaths. Motor vehicles are responsible for the overwhelming majority of "accidental" deaths in the United States. In 1992, more than 4,800 teens aged 15 to 19 years died because of injuries from motor vehicle accidents. Death rates associated with motor vehicle crashes rise more rapidly in adolescence than at any other age. The death rate for 15- to 19-year-olds from motor vehicle injuries is 28 per 100,000. This is 10 times higher than the death rate from motor vehicle injuries in the 10- to 14-year-old age group. Since 1970, the motor vehicle death rate for male adolescents has declined, and the rate for female adolescents has increased slightly. The death rate for boys, however, remains twice that for girls. Most teens who die due to a motor vehicle-related injury are occupants of the vehicle, and the most common cause of death is a head injury. Male adolescents are at greater risk than females in every category of motor vehicle crashes. Race also is an important risk factor, with the death rate for Native Americans being more than twice that for whites and blacks. Impairment due to alcohol is an important behavioral factor contributing to the increase in motor vehicle deaths during later adolescence. Approximately 40% of the adolescent drivers involved in fatal motor vehicle crashes in 1991 tested positive for alcohol use at the time of the injury, even though none was old enough to drink legally. The second most common type of unintentional injury leading to death in adolescence is drowning, which also often is related to alcohol use, and which was associated with a death rate of 2.3 per 100,000 teens aged 15 to 19 in 1992. This is one-tenth the death rate from motor vehicle injuries. Homicide has become the leading cause of death for black adolescent males in our country and the second most common cause of death among teenagers. In 1991, more than 26,500 Americans were victims of homicide. Adolescents and young adults (15 to 24 years old) comprised approximately one third of this total. Thus, 22 young people are murdered in this country every dayalmost one per hour. Firearms are implicated in 68% to 75% of all homicides; knives are used in 18% to 25%. Most of the increase in homicide rates since 1985 can be attributed to the increase in firearm-related deaths, 73% of which were due to handguns. Over the past 30 years, the suicide rate among adolescents has tripled from approximately 4 per 100,000 to 12 per 100,000. Suicide is the cause of death for more than 5,000 teens in the United States each year. This increase in suicide deaths has been most pronounced for white males. Nonfatal suicidal behavior also represents a major cause of medical and psychiatric morbidity in this age group. The number of high school students who have attempted suicide has been reported to be as high as 9%. Girls are four times more likely than boys to attempt suicide, but boys are four times more likely than girls to die

from a suicide attempt. This difference probably is due primarily to the specific suicide method attempted. Boys are ten times more likely than girls to use a firearm and three times more likely than girls to use hanging for suicide attempts. In addition to gender factors, the risk of suicide varies with race and ethnicity. Whites and Native Americans are at higher risk than blacks and Hispanics. Additionally, adolescents living in the Western states and rural areas are at higher risk than other adolescents. Studies of adolescents who died from suicide have shown that as many as 50% had made previous attempts. References: Brindis CD, Irwin CE Jr, Millstein SG. United States profile. In: McAnarney ER, Kreipe RE, Orr DP, Comerci GD, eds. Textbook of Adolescent Medicine. Philadelphia, Pa: WB Saunders Co; 1992:12-27 Cohall AT, Cohall RM. "Number one with a bullet" epidemiology and prevention of homicide among adolescents and young adults. Adolescent Medicine: State of the Art Reviews. 1995;6:183-197 Kachur SP, Potter LB, Powell KE, Rosenberg ML. Suicide: epidemiology, prevention, treatment. Adolescent Medicine: State of the Art Reviews. 1995;6:171-182 Li G, Baker SP, Frattaroli S. Epidemiology and prevention of traffic-related injuries among adolescents. Adolescent Medicine: State of the Art Reviews. 1995;6:135-151 Setterberg SR. Suicidal behavior and suicide. In: Friedman SB, Fisher M, Schonberg SK, eds. Comprehensive Adolescent Health Care. St Louis, Mo: Quality Medical Publishing, Inc; 1992:862-867 Critique 127 Preferred Response: A

[View Question] Puberty usually begins after age 8 in girls. The first sign of secondary sexual development in 90% of girls is thelarche or breast budding, which occurs at an average age of 11 years. Lack of breast development by age 13 indicates delayed development and requires evaluation to determine the cause. Pubarche (the development of pubic hair) usually follows breast budding by a few months. Breast and pubic hair development generally continue in parallel, so that the stages of sexual maturity are rarely more than one stage apart in girls. If breast development reaches stage 5 (adult) without pubic hair development, the possibility of androgen insensitivity (testicular feminization) should be considered. Conversely, lack of breast development when pubic hair development is complete suggests an excess of androgens, a lack of estrogen, or a congenital absence of breast tissue. Menarche usually occurs 2 years after thelarche, when breast development is at sexual maturity stage 3 or stage 4. The average age of menarche in the United States is about 12 years and 9 months. Early menses are often short (2 to 3 days), with longer intervals of 40 to 60 days between cycles.

Linear growth in the 4 years prior to the onset of puberty is fairly constant at approximately 4 cm per year. During the first 2 years of female puberty, the growth rate increases by 25% to 75%, reaching a peak velocity of 8 to 10 cm per year. This peak growth velocity precedes menarche by about 1.3 years. By menarche, the growth rate has decreased substantially; most girls are within 4 cm of their adult height. Girls who mature early tend to have a higher peak growth rate compared with girls who mature later, but they have a shorter final adult height. References: Copeland KC. Variations in normal sexual development. Pediatrics in Review. 1986;8:47-55 Kreipe RE. Normal somatic adolescent growth and development. In: McAnarney ER, Kreipe RE, Orr DP, Comerci GD, eds. Textbook of Adolescent Medicine. Philadelphia, Pa: WB Saunders Co; 1992:44-67 Kulin HE. Normal pubertal development. In: Rudolph AM, Hoffman JIE, Rudolph CD, eds. Rudolph's Pediatrics. 20th ed. Stamford, Conn: Appleton & Lange; 1996:1790-1794 Marshall WA, Tanner JM. Variations in pattern of pubertal changes in girls. Arch Dis Child. 1969;44:291-303 Marshall WA, Tanner JM. Variations in the pattern of pubertal changes in boys. Arch Dis Child. 1970;45:13-23 Needlman RD. Adolescence. In: Nelson WE, Behrman RE, Kliegman RM, Arvin AM, eds. Nelson Textbook of Pediatrics. 15th ed. Philadelphia, Pa: WB Saunders Co; 1996:58-62 Schneider MB. Physical examination. In: Friedman SB, Fisher M, Schonberg SK, eds. Comprehensive Adolescent Health Care. St Louis, Mo: Quality Medical Publishing, Inc; 1992:58-69 Critique 147 Preferred Response: A

[View Question] Coronary heart disease (CHD) is the major cause of death in the United States. Elevated levels of blood cholesterol are strongly implicated in the pathogenesis of CHD, and there is compelling evidence that CHD begins in childhood and progresses slowly into adulthood. Indeed, postmortem studies have demonstrated the presence of coronary atherosclerosis as early as the second decade of life. Routine blood cholesterol screening of all adolescents remains controversial. In general, screening in the context of well adolescent care is recommended for those who have a family history of premature (<55 years of age) cardiovascular disease or high cholesterol levels. Thus, it probably is appropriate to measure levels in the boy in the vignette at the present time rather than waiting until he is an adult. Some experts also recommend screening adolescents who have other risk factors for cardiovascular disease, including hypertension, smoking, physical inactivity, or high-fat diets.

Hypercholesterolemia may be idiopathic or related to medical disorders, including diabetes mellitus, hypothyroidism, hepatitis, obstructive liver disease, nephrosis, systemic lupus erythematosus, and anorexia nervosa. Certain drugs (eg, steroids, anticonvulsants, thiazides, isotretinoin) also can raise cholesterol levels. Cholesterol levels vary with both age and pubertal status. Adolescents older than age 12 years have lower cholesterol levels than children aged 9 to 12 years. In general, cholesterol levels obtained in infancy correlate poorly with those obtained during childhood years. In childhood, the average total cholesterol level is approximately 150 mg/dL. Additionally, adolescents at Tanner stage or sexual maturity rating (SMR) 5 have a mean cholesterol level 10 mg/dL lower than adolescents who have a SMR of 1 to 2. Girls have higher cholesterol levels than boys in all age groups. Increased body mass index, decreased physical activity, and a high-fat diet are associated with elevated cholesterol levels. When interpreting cholesterol levels, it is important to realize that truly "normal" values are unknown. The expert panel of the National Cholesterol Education program (NCEP) has classified total cholesterol levels for children 10 to 19 years as acceptable (<170 mg/dL), borderline (170 to 199 mg/dL), and high (>200 mg/dL). Thus, a level of approximately 210 mg/dL would not be considered appropriate for the boy in the vignette. The variability of cholesterol levels and difficulty in determining "normal" values is illustrated by a study of more than 600 children between the ages of 3 and 19 years, in which only 40% of those who had high cholesterol levels continued to have elevated concentrations at follow-up. Total cholesterol levels of less than 170 mg/dL do not require any intervention. If the total cholesterol level is high (>200 mg/dL), the blood test should be repeated because there is substantial day-to-day variation in cholesterol values in a single individual. Therefore, an adolescent should be fasting, in his or her usual state of health, and have at least two cholesterol levels measured before it is determined that the level truly is elevated. The management of adolescents who have high cholesterol levels of 200 mg/dL or greater is controversial, but principles that are followed by many include: 1) Diagnose and treat secondary causes of hypercholesterolemia, 2) Reduce risk factors by intervening with those that can be altered (eg, smoking, hypertension), and 3) Recommend a heart healthy diet with decreased amounts of fat. References: Benuck I, Gidding SS, Donovan M. Year-to-year variability of cholesterol levels in a pediatric practice. Arch Pediatr Adolesc Med. 1995;149:292-296 Jacobson MS. Hyperlipidemia and atherosclerosis. In: Friedman SB, Fisher M, Schonberg SK, eds. Comprehensive Adolescent Health Care. St Louis, Mo: Quality Medical Publishing, Inc; 1992:213-216 Kwiterovich PO Jr. Disorders of lipid and lipoprotein metabolism. In: Rudolph AM, Hoffman JIE, Rudolph CD, eds. Rudolph's Pediatrics. 20th ed. Stamford, Conn: Appleton & Lange; 1996:343-347

Primrose ED, Savage JM, Boreham CA, Carn GW, Strain JJ. Cholesterol screening and family history of vascular disease. Arch Dis Child. 1994;71:239-242 Rocchini AP. Cardiovascular risk factors and prevention. In: McAnarney ER, Kreipe RE, Orr DP, Comerci GD, eds. Textbook of Adolescent Medicine. Philadelphia, Pa: WB Saunders Co; 1992:365-381 Sanchez-Bayle M, Gonzalez-Requejo A, Baeza J, et al. Diet therapy for hypercholesterolemia in children and adolescents. A follow-up. Arch Pediatr Adolesc Med. 1994;148:28-32 Starc TJ, Deckelbaum RJ. Evaluation of hypercholesterolemia in childhood.03572 Pediatrics in Review. 1996;17:94-98 Critique 169 Preferred Response: A

[View Question] The sequence of somatic changes that occur during puberty forms the basis of the Tanner staging or sexual maturity rating (SMR), which classifies pubertal development into five stages. Although there is marked individual variation in both the age of onset and the duration of puberty, the sequence of changes is similar in all boys. The SMR is determined by examining the scrotum, testicles, penis, and pubic hair. There may be a difference of one stage between gonadal, pubic hair, and penile development. Thus, a SMR of 4 for pubic hair and 3 for penile growth is within the normal variation and does not require further evaluation. The first physical sign of pubertal development in boys is testicular enlargement; testicular length greater than 2.5 cm is consistent with gonadal stimulation and the onset of puberty. This aspect of pubertal development usually occurs between 10 and 11 years of age; 97% of boys will have a testicular length greater than 2.5 cm by age 14 years. The penis normally does not begin to enlarge until the testes are SMR 3. Similarly, pubic hair usually does not appear until approximately 1 year after the testicles have begun to enlarge. The discrepancy between testicular size and other secondary sexual characteristics in the boy in the vignette is suggestive of a nontesticular source of androgen causing penile enlargement and pubic hair development. The peak height velocity for boys usually is reached at age 13.5 years. Thus, by 14 years most boys will have a decreasing height velocity. The normal range of height velocity for 14-year-old boys is 6 to 11 cm/y. Accordingly, a boy who is growing at 7 cm/y, like the one in the vignette, is within the normal range. The growth spurt in boys generally lags behind girls by 2 years, but because boys continue to grow for a prolonged period after they have attained peak height velocity, their ultimate adult height is greater than that of girls. Axillary hair generally does not appear until after pubic hair development has reached SMR 5. Facial hair typically is not noticeable until late puberty. It develops in an ordered sequence, beginning first on the outer corners of the upper lip, then the upper cheeks and middle of the lower lip, and followed finally along the sides and lower border of

the chin. Body hair on the chest and back develops gradually after facial hair. Thus, it is not abnormal for a boy who is 14 years old and has SMR 3 to 4 development to lack facial hair. Approximately 60% of boys have noticeable breast development (gynecomastia) in midpuberty when their genital and pubic hair development is SMR 3. The enlargement is usually bilateral, subareolar, and averages 20 to 30 mm in diameter. The breast tissue usually regresses in 1 to 2 years, requiring no intervention. However, persistent gynecomastia may be a sign of testicular failure that requires further endocrinologic evaluation. If breast enlargement persists into late puberty, cosmetic surgery may be considered if the enlargement is a source of embarrassment for the boy. References: Copeland KC. Variations in normal sexual development. Pediatrics in Review. 1986;8:47-55 Kreipe RE. Normal somatic adolescent growth and development. In: McAnarney ER, Kreipe RE, Orr DP, Comerci GD, eds. Textbook of Adolescent Medicine. Philadelphia, Pa: WB Saunders Co; 1992:44-67 Kulin HE. Normal pubertal development. In: Rudolph AM, Hoffman JIE, Rudolph CD, eds. Rudolph's Pediatrics. 20th ed. Stamford, Conn: Appleton & Lange; 1996:1790-1794 Marshall WA, Tanner JM. Variations in the pattern of pubertal changes in boys. Arch Dis Child. 1970;45:3-23 Needlman RD. Adolescence. In: Nelson WE, Behrman RE, Kliegman RM, Arvin AM, eds. Nelson Textbook of Pediatrics. 15th ed. Philadelphia, Pa: WB Saunders Co; 1996:58-62 Schneider MB. Physical examination. In: Friedman SB, Fisher M, Schonberg SK, eds. Comprehensive Adolescent Health Care. St Louis, Mo: Quality Medical Publishing, Inc; 1992:58-69 Critique 189 Preferred Response: E

[View Question] One of the most helpful clues to the diagnosis of anorexia nervosa is a distorted body image. Almost all adolescents who have anorexia will describe themselves as fat, despite being underweight. Indeed, this disturbance of body perception is one of the four major criteria used by The American Psychiatric Association to diagnose anorexia nervosa. The other criteria are: 1) weight that is at least 15% below expected weight for height or failure to make expected weight gain; 2) intense fear of gaining weight or of becoming fat, even though underweight; and 3) in females, absence of at least three consecutive menstrual cycles when otherwise expected to occur, ie, primary or secondary amenorrhea. A history of dieting alone is not especially helpful in distinguishing between anorexia nervosa and other causes of weight loss because more than 60% of all adolescent females in the United States diet at some time to lose weight. However, girls who persist in dieting, even after achieving their initial weight goal, may be exhibiting early signs of anorexia

nervosa. Also, adolescents whose weight goal is inappropriate for their height may be at risk for developing anorexia. Adolescents who have anorexia nervosa often participate in athletic activities and exercise regularly. In some cases, these teenagers will exercise excessively to lose weight or to control their feelings of hunger. Many healthy adolescents, however, exercise for 2 hours per day, especially during sport seasons. Exercise should be considered excessive when it is pursued at the exclusion of other social activities or against medical advice during recovery from injuries or illnesses. At times it may be difficult to distinguish between anorexia nervosa and compulsive, excessive exercise because both conditions are associated with weight loss and amenorrhea. However, the adolescent who has anorexia nervosa has a disordered body image and feels fat, even if she is underweight. This contrasts with the body image of the psychologically healthy athlete who recognizes herself as thin and does not have an intense fear of gaining weight. When seriously underweight, many adolescents who have anorexia nervosa manifest depressive symptoms, including depressed mood, social withdrawal, irritability, and insomnia. Such symptoms have been observed in other patients who are undergoing starvation; they may be physiologic sequelae of semistarvation. Thus, it is important to reassess the patient who has mood disturbance and anorexia after weight is restored to determine if the depressive symptoms are persisting. Major depressive disorder is a frequent comorbidity in adolescents who have eating disorders such as anorexia nervosa. The adolescent who has anorexia nervosa loses weight primarily by reducing total food intake. Anorexia nervosa has been subdivided into two eating types: restrictive and binge eating/purging. Patients in the restrictive subtype diet, fast, and exercises excessively. The other subtype of patients regularly engage in binge eating, purging, or both. Purging may be accomplished by self-induced vomiting or the misuse of laxatives, diuretics, or enemas. Some individuals in this subtype do not binge eat, but nonetheless purge after eating even small quantities of food. Over-the-counter medications such as diet pills, laxatives, and diuretics frequently are taken by teens trying to lose weight who do not have an eating disorder. Because such behaviors are so common, the use of diuretics to control weight is not very helpful in distinguishing between the adolescent who is psychologically healthy and one who has anorexia nervosa. References: Coupey SM. Anorexia nervosa. In: Friedman SB, Fisher M, Schonberg SK, eds. Comprehensive Adolescent Health Care. St Louis, Mo: Quality Medical Publishing, Inc; 1992:217-231 Irwin CE Jr, Ryan SA. The adolescent patient. In: Rudolph AM, Hoffman JIE, Rudolph CD, eds. Rudolph's Pediatrics. 20th ed. Stamford, Conn: Appleton & Lange; 1996:43-44 Jay MS, Morse P, Rickert VI. Who is the customer? Adolescent Medicine: State of the Art Reviews. 1996;7:389-393 Lehman RM. Poorly controlled diabetes? Adolescent Medicine: State of the Art Reviews. 1996;7:405-408

Litt IF. Anorexia nervosa and bulimia. In: Nelson WE, Behrman RE, Kliegman RM, Arvin AM, eds. Nelson Textbook of Pediatrics. 15th ed. Philadelphia, Pa: WB Saunders Co; 1996:549550 Nussbaum MP. Anorexia nervosa. In: McAnarney ER, Kreipe RE, Orr DP, Comerci GD, eds. Textbook of Adolescent Medicine. Philadelphia, Pa: WB Saunders Co; 1992:536-541 Critique 211 Preferred Response: A

[View Question] Irregular, heavy menstrual bleeding (menometrorrhagia) in the young adolescent most often is due to anovulation and immaturity of the hypothalamic-pituitary-ovarian axis. When ovulation does not occur, there is prolonged unopposed stimulation of the endometrium, which becomes hyperplastic but fragile and friable. This leads to spontaneous bleeding that is acyclic. However, within 2 years of menarche, most girls will develop regular ovulatory cycles that occur every 21 to 45 days. Failure to establish regular cycles within 2 years of menarche suggests that the adolescent described in the vignette has another cause for her heavy menses. Blood dyscrasias are an uncommon cause of menometrorrhagia, but they should be considered when heavy menstrual bleeding is associated with prolonged bleeding from minor surgeries or injuries. Adolescents who have hereditary bleeding disorders such as von Willebrand disease often have severe menorrhagia that begins at menarche. In contrast, adolescents who have menorrhagia due to anovulatory cycles usually have irregular but light flow for the first 6 months after menarche and only develop severe menorrhagia after prolonged anovulation and unopposed estrogen stimulation of the endometrium. The amount of blood loss in a normal menstrual period ranges from 40 to 60 mL, but this may be difficult to quantify. The differential diagnosis of heavy menses also includes endocrine disorders (eg, hypothyroidism). Although menstrual bleeding is a common cause of mild anemia in adolescence, a hemoglobin concentration as low as 8 g/dL never should be attributed to normal, physiologic menses. Other causes of menometrorrhagia include pelvic inflammatory disease and ectopic pregnancy. Both of these diseases will cause heavy, painful menstrual flow, but neither would be expected to cause persistent menorrhagia beginning with the onset of menses. Tubal pregnancy usually is associated with a positive pregnancy test. Trauma also can cause abnormal, heavy menstrual flow. The adolescent who has been sexually assaulted often is reluctant to tell her family and physician about the assault and may seek medical care only when complications occur. Thus, it is imperative for physicians to consider the possibility of assault whenever a woman presents with unexplained injuries. The presence of a hand-shaped bruise on the patient in the vignette should alert the physician to the possibility of domestic violence. Unfortunately, many adolescent girls are victims of violent acts perpetrated by their boyfriends. Often they are reluctant to tell family and friends about these episodes, and they become caught in a cycle

of violence and fear. Whenever violence is suspected, patients should be provided with information to help them remove themselves from the situation. References: Litt IF. Menstrual problems. In: Nelson WE, Behrman RE, Kliegman RM, Arvin AM, eds. Nelson Textbook of Pediatrics. 15th ed. Philadelphia, Pa: WB Saunders Co; 1996:554-556 Moscicki A-B. Common menstrual problems. In: Rudolph AM, Hoffman JIE, Rudolph CD, eds. Rudolph's Pediatrics. 20th ed. Stamford, Conn: Appleton & Lange; 1996:60-63 Polaneczky MM, Slap GB. Menstrual disorders in the adolescent: dysmenorrhea and dysfunctional uterine bleeding.01195 Pediatrics in Review. 1992;13:83-87 Purcell JS, Hergenroeder AC. Severe menorrhagia. Adolescent Medicine: State of the Art Reviews. 1996;7:449-454 Ritter DB, Rickard C, Rivera M, Alderman EM. Lower abdominal pain in a perimenarchal adolescent. Adolescent Medicine: State of the Art Reviews. 1996;7:455-459 Sanfilippo JS, Hertweck SP. Menstruation. In: Friedman SB, Fisher M, Schonberg SK, eds. Comprehensive Adolescent Health Care. St Louis, Mo: Quality Medical Publishing, Inc; 1992:917-946 Section on Child Abuse and Neglect. Domestic violence. In: A Guide to References and Resources in Child Abuse and Neglect. Elk Grove Village, Ill: American Academy of Pediatrics; 1994:16-17 Wenning JB. Dysfunctional uterine bleeding. In: McAnarney ER, Kreipe RE, Orr DP, Comerci GD, eds. Textbook of Adolescent Medicine. Philadelphia, Pa: WB Saunders Co; 1992:666-671 Critique 232 Preferred Response: A

[View Question] In 1969, Marshall and Tanner recorded the sequence of pubertal development for girls and boys. Sexual maturation was assessed by examining the breasts and pubic hair in girls and the genitals and pubic hair in boys. The resulting rating divides pubertal development into five stages. Prepubertal children who have no signs of sexual maturity are described as having a sexual maturity rating (SMR) of stage 1; adolescents who have achieved full sexual maturation are in stage 5. The time interval from stage 2 to stage 5 in both girls and boys is approximately 4.5 years, with a range of 3 to 6 years. In approximately 90% of girls, the first visible sign of pubertal development is breast budding. Stage 2 of breast development is the breast bud stage in which there is a slight elevation of the breast and an increase in the diameter of the areola. Stage 3 breast development is characterized by further enlargement of the breast and areola with no separation of the contours. In stage 4, there is further enlargement of the breast with

projection of the areola and nipple to form a secondary mound above the level of the breast. Stage 5 is mature breast. In stage 2 of pubic hair development, girls have sparse growth of straight hair along the labia. By stage 3, the pubic hair is thicker and has spread to the junction of the pubis, and by stage 4 the pubic hair becomes similar to adult hair and has spread over the mons pubis. By stage 5, the pubic hair has extended onto the thighs. Typically, breast and pubic hair developmental stages run parallel in girls; that is, when breasts are stage 2 in development, the pubic hair is also stage 2. On the other hand, the stage of genital development in boys usually is more advanced than the stage of pubic hair development. Thus, it is normal for boys to have genital development at stage 3 when pubic hair is only stage 2. In boys, the first sign of puberty is enlargement of the testes and scrotum. This is followed by penile enlargement and pubic hair development, both of which are due to increased testosterone production by the testes. Stage 2 genital development in boys is characterized by enlargement of testes, scrotal reddening, and increased rugae. In Stage 3, the penis increases in length and breadth and the testes grow further. The scrotal skin continues to darken in Stage 4, and there is further increase in size of the testes and penis. Stage 5 is mature development. Gynecomastia or breast development in boys usually occurs when the genital development is stage 3. This physical finding is quite common in boys during midpuberty, but is not part of the SMR system. Darkening of the areolae in girls occurs with pregnancy and also is not included in the physical assessment of sexual maturity. References: Copeland KC. Variations in normal sexual development. Pediatrics in Review. 1986;8:47-55 Kreipe RE. Normal somatic adolescent growth and development. In: McAnarney ER, Kreipe RE, Orr DP, Comerci GD, eds. Textbook of Adolescent Medicine. Philadelphia, Pa: WB Saunders Co; 1992:44-67 Kulin HE. Normal pubertal development. In: Rudolph AM, Hoffman JIE, Rudolph CD, eds. Rudolph's Pediatrics. 20th ed. Stamford, Conn: Appleton & Lange; 1996:1790-1794 Marshall WA, Tanner JM. Variations in pattern of pubertal changes in girls. Arch Dis Child. 1969;44:291-303 Marshall WA, Tanner JM. Variations in the pattern of pubertal changes in boys. Arch Dis Child. 1970;45:13-23 Needlman RD. Adolescence. In: Nelson WE, Behrman RE, Kliegman RM, Arvin AM, eds. Nelson Textbook of Pediatrics. 15th ed. Philadelphia, Pa: WB Saunders Co; 1996:58-63

Schneider MB. Physical examination. In: Friedman SB, Fisher M, Schonberg SK, eds. Comprehensive Adolescent Health Care. St Louis, Mo: Quality Medical Publishing, Inc; 1992:58-69 Critique 254 Preferred Response: D

[View Question] The differential diagnosis of lower abdominal pain in the sexually active adolescent female always should include gynecologic conditions such as ovarian disorders, sexually transmitted diseases, pregnancy, or pregnancy complications. Accordingly, a pelvic examination is a very important part of the clinical evaluation. A speculum examination will allow the physician to examine the cervix and vaginal mucosa. Cervical specimens should be obtained for culture or other rapid diagnostic tests (eg, antigen detection, DNA probe, polymerase chain reaction, ligase chain reaction) for both Neisseria gonorrhoeae and Chlamydia trachomatis. In addition, if a cervical or vaginal discharge is noted, microscopic examination of the discharge may be helpful. Both a potassium hydroxide (KOH) and a normal saline slide should be examined. The KOH slide will be helpful in identifying hyphae suggestive of Candida infection, while the normal saline slide can aid in identifying bacteria and trichomonads. A bimanual examination also should be performed to determine adnexal size and tenderness as well as cervical motion tenderness. If a sexually active girl who has lower abdominal pain also exhibits adnexal tenderness and cervical motion tenderness on pelvic examination, a presumptive diagnosis of pelvic inflammatory disease (PID) can be made. Indeed, studies have shown that more than 60% of adolescent girls who have these three findings will have laparoscopically confirmed PID. Other laboratory and clinical features that can be helpful in diagnosing PID include an elevated erythrocyte sedimentation rate, fever, and palpable adnexal swelling. If any one of these additional features is present, the probability of confirming the diagnosis of PID on laparoscopy increases to 78%. If two of these features are present, the likelihood of diagnosing PID increases to 90%, and if all three are present, the potential for PID increases to 96%. Age is one of the most important risk factors for the development of PID. Indeed, the risk of developing PID is 1 in 8 for sexually active girls between the ages of 15 and 19 compared with approximately 1 in 80 for girls 25 to 29 years of age. Because no single clinical finding or laboratory test is diagnostic of PID, the physician must maintain a high index of suspicion whenever an adolescent girl complains of lower abdominal pain. The differential diagnosis of lower abdominal pain also includes urinary tract infection. Sexually active girls are more likely to develop cystitis than virginal girls, especially soon after the onset of sexual activity. However, this diagnosis is unlikely if the urinalysis is normal, such as in the girl in the vignette. Pelvic ultrasonography may help to determine the cause of lower abdominal pain in the sexually active adolescent. However, a pelvic examination should precede pelvic ultrasonography. If results of the pelvic examination are normal, ultrasonography usually is

not indicated because most gynecologic problems that can be diagnosed by ultrasonography are associated with clinically significant findings on pelvic examination, such as adnexal masses, tenderness, or cervical motion tenderness. However, because a pelvic examination may be uncomfortable for the virginal adolescent, pelvic ultrasonography may be the test preferred by both the adolescent and her parent. Even in the virginal adolescent, a careful examination of the external genitalia always is indicated. Abdominal radiography is sometimes helpful in the diagnosis of lower abdominal pain, especially if acute appendicitis is suspected. However, it is always important to check a pregnancy test prior to obtaining abdominal radiographs in the sexually active adolescent who has had unprotected sex. Pancreatitis is an uncommon cause of abdominal pain in adolescence. Thus, measurement of serum amylase activity is rarely helpful in the evaluation of the adolescent who has lower abdominal pain. Because adolescents who have systemic lupus erythematosus, Crohn disease, diabetes mellitus, and hyperlipidemia are at increased risk for pancreatitis, a serum amylase may be indicated if there is a history of any of these disorders. There also are a number of infectious agents and toxins, especially alcohol, that may cause pancreatitis in adolescents. Thus, a careful drug history should be part of the evaluation of the adolescent who has abdominal pain, especially if pancreatitis is suspected. References: Litt IF. Menstrual problems. In: Nelson WE, Behrman RE, Kliegman RM, Arvin AM, eds. Nelson Textbook of Pediatrics. 15th ed. Philadelphia, Pa: WB Saunders Co; 1996:554-556 Moscicki A-B. Common menstrual problems. In: Rudolph AM, Hoffman JIE, Rudolph CD, eds. Rudolph's Pediatrics. 20th ed. Stamford, Conn: Appleton & Lange; 1996:60-63 Polaneczky MM, Slap GB. Menstrual disorders in the adolescent: dysmenorrhea and dysfunctional uterine bleeding.01210 Pediatrics in Review. 1992;13:83-87 Ritter DB, Rickard C, Rivera M, Alderman EM. Lower abdominal pain in a perimenarchal adolescent. Adolescent Medicine: State of the Art Reviews. 1996;7:455-459 Rosenfeld WD, Litman N. Sexually transmitted diseases. In: Friedman SB, Fisher M, Schonberg SK, eds. Comprehensive Adolescent Health Care. St Louis, Mo: Quality Medical Publishing, Inc; 1992:995-1015 Shafer M-A. Sexually transmitted disease syndromes. In: McAnarney ER, Kreipe RE, Orr DP, Comerci GD, eds. Textbook of Adolescent Medicine. Philadelphia, Pa: WB Saunders Co; 1992:696-710 Wald ER. Pelvic inflammatory disease in adolescents. Current Problems in Pediatrics. 1996;26:86-97

You might also like